2032 final Flashcards

1
Q

Quelles sont les 3 formes de vitamines K

et leur origine (végétal, animal, synthetique bacterien, endogene etc.)

Leur numéro: exemple : k1 , k2 etc.

A

Phylloquinone
(K1) Règne végétal

Menaquinone-n
(K2)
Origine Bactérienne
Production endogène

Menadione
(K3)
Forme
synthétique

How well did you know this?
1
Not at all
2
3
4
5
Perfectly
2
Q

Comment la vitamine K est-elle digérée et absorbée?
Indices:
Digestion: foie et pancréas
Absorption: endroit et mécanisme d’absorption.
enterocytes

A

La digestion nécéssite les sels biliares et l’incorporation aux mycelles, on a aussi besoin des enzymes pancréatiques lipases et phospholipases. Elle est ensuite absorbée de façon passive de diffusion dans le jénunum et l’iléon, puis est incorporée au chylomichrons.

How well did you know this?
1
Not at all
2
3
4
5
Perfectly
3
Q

Quelles sont les principales sources de Phylloquinones?

A
Légumes verts
Þ 45-60% VK quotidienne total
• Huiles (canola, soya, olive)
Þ huiles et plats composés
~30% VK quotidienne totale
• Légumineuses (soya, haricot mungo,
lentilles)
• Fines herbes, épices, certaines noix
(pistache)
• Certaines algues (hijiki, kombu)
How well did you know this?
1
Not at all
2
3
4
5
Perfectly
4
Q

Quelles sont les principales sources de ménaquinones?

Pourquoi les produits fermentés en contiennent?

A
Ménaquinones
• Produits laitiers
• Viande, volaille
Nato contient bcp de menaquinone.
Fromage est aussi un produit fermenté et donc bcp de menaquinone

Origine bactérienne des ménaquinones.

How well did you know this?
1
Not at all
2
3
4
5
Perfectly
5
Q

Quelle est la biodisponibilité des différentes formes de phylloquinones?

A

Phylloquinones 15-20%

Phylloquinone
pur 80%

How well did you know this?
1
Not at all
2
3
4
5
Perfectly
6
Q

Quelle est la biodisponibilité des ménaquinones? Comment sait on qu’elle est absorbée?

A

On ne sait pas trop, mais celles de production endogene serait de l’ordre de 0,3-5,1 mg MK (surtout Mk-9), absoprtion serait minime.

on en retrouve dans le sang et dans le foie.

How well did you know this?
1
Not at all
2
3
4
5
Perfectly
7
Q

Décrivez le transport, la distribution et l’Excértion de la vitamine K dans le corps. (réponse longue - slide 6 du doc)

A

Le chylomichron transporte la vit K dans la foie lymphatique, les résidus de CM vont au foie et la vit K finie a l’hepatocyte.

De l’hepatocyte, soit qu’elle est excrétée via la bile puis les celles, mais peut etre réabsorbées par la boucle enterohepatique.

Sinon l’hepatocte peut la stocker.

Finalement l’hepatocyte peut faire un VLDL ou la vit K est intégrée et va aller aux tissus cibles.

How well did you know this?
1
Not at all
2
3
4
5
Perfectly
8
Q

Quelle est la principale voie d’utilisation de la vit K?

A

La gamma carboxylation

How well did you know this?
1
Not at all
2
3
4
5
Perfectly
9
Q

Décrivez le cycle de la vitamine K dans le but de la gamma carboxylation.

A

Quinone vient de l’alimentation, elle est hydrolyser pour faire l’hydroquinone. Elle devient cofacteur de la réaction de carboxylation des résidus Glu en résidus Gla. Elle devient ensuite l’epoxyde une fois utilisée comme cofacteur et sera recyclée sous la forme quinone.

How well did you know this?
1
Not at all
2
3
4
5
Perfectly
10
Q

Vit K:

A quoi sert la carboxylation?

A

Permet aux protéines d’etre fonctionelle biologiquement.

How well did you know this?
1
Not at all
2
3
4
5
Perfectly
11
Q

Pourquoi dépent on peut de l’alimentation pour la vitamine K?

A

La vit K est recyclée dans son cycle comme cofacteur de la carboxylation, qui est la fonction principale.

How well did you know this?
1
Not at all
2
3
4
5
Perfectly
12
Q

Nommez 2 types de protéines les plus fréquentes qui ont besoin de la vit K et de son action de cofacteur de carboxylation des résidus glu en résidus gla

A

Surtout coagulation puis métabolisme osseux.

How well did you know this?
1
Not at all
2
3
4
5
Perfectly
13
Q

Associez les bonnes protéines qui ont besoin de la vit K aux bonnes fonctions.

Prothrombine
Facteur IX
protéine C
MGP
Gas6   
Ostéocalcine
inhibiteur calcification
régulateur cycle cellulaire
formation osseuse
procoagulant (2 fois)
anticoagulant
A
Prothrombine procoagulant
Facteur IX procoagulant
Protéine C anticoagulant
Ostéocalcine formation osseuse
MGP inhibiteur calcification
Gas6 régulateur cycle cellulaire
How well did you know this?
1
Not at all
2
3
4
5
Perfectly
14
Q

Autre que la carboxylation, la vit K a aussi d’autres fonctions importantes associez a un metabolite important de la vit K autre que l’hydroquinone.

Nommez ce métabolite, donnez son numero (ex K1, k2),

décrivez comment il est formé

nommez les fonctions de ce métabolite (3 fonctions).

A

L’organisme transforme la phylloquinone en
ménaquinone-4 (MK-4)

La MK-4 pourrait agir au plan physiologique
indépendamment des protéines VKdépendantes
– Synthèse des sphingolipides
– Protège contre le stress oxydatif
– Action anti-inflammatoire

How well did you know this?
1
Not at all
2
3
4
5
Perfectly
15
Q

Quelles sont les 3 types de carences principales associez a la vitamine K?

A

Apports, absoprtion, fonctions

How well did you know this?
1
Not at all
2
3
4
5
Perfectly
16
Q

Donnez 2 exemples de carences en vit K associée aux apports

A

Rare
(alim parentérale)

Thérapie
antibiotiques

How well did you know this?
1
Not at all
2
3
4
5
Perfectly
17
Q

Donnez 4 exemples de carences en vit K associée a l’absoprtion

A

Maladie coeliaque
Abêtalipoprotéinémie

Fibrose kystique

Cholestase
Maladies hépatiques:
Cirrhose, hépatite…

Pancréatite

How well did you know this?
1
Not at all
2
3
4
5
Perfectly
18
Q

Donnez 1 exemples de carences en vit K associée aux fonctions de la vit K

A

Thérapie

Anticoagulants

How well did you know this?
1
Not at all
2
3
4
5
Perfectly
19
Q

Comment la warfarine impacte le metabolisme de la vit K?

A

Bloque l’enzyme qui permet l’hydroxylation de la quinone et donc empeche notamment la coagulation.

How well did you know this?
1
Not at all
2
3
4
5
Perfectly
20
Q

Pourquoi le nouveau né est a risque de carence en vit K? (4 causes)

Quelle complication cela peut entrainer?

COmment prévient on cette complication?

A

-Intestin stérile
-Lait maternel
pauvre en vit.K
-Pas de réserves
-Niveaux faibles
des Fact. coagulation

hémoragie

Injection
intra-musculaire
prophylaxique:
1ères heures de vie

How well did you know this?
1
Not at all
2
3
4
5
Perfectly
21
Q

Quelles sont les 2 recommandations nutritionnelles pour les personnes qui suivent une therapie anticoagulants a la warfarine?

A

Tenter de maintenir des apports de VK constants
d’une journée à l’autre
• Des apports élevés (>400 UI/J) de vitamine E
pourraient entraîner des saignements chez
personnes anticoagulées (donc a éviter

How well did you know this?
1
Not at all
2
3
4
5
Perfectly
22
Q

Quels sont les marqueurs de l’état nutritionnel en vit K? Lesquels sont utilisé en clinqiue et en recherche ou les 2?

A

Temps de coagulation (INR: International
normalized ratio, PT, APTT) clinique

Facteurs de coagulation (Facteur VII) clinique

VK plasmatique Þ clinique/recherche
PIVKA-prothrombine Þ recherche
Ostéocalcine sous-carboxylée (PIVKA II)

How well did you know this?
1
Not at all
2
3
4
5
Perfectly
23
Q

Vit K
Pour le Temps de coagulation (INR: International
normalized ratio, PT, APTT), discuter de la sensibilité de ce marqueur.

A

Pas sensible, reflete un cas tres avancé de carence.

How well did you know this?
1
Not at all
2
3
4
5
Perfectly
24
Q

Quelle est l’utilité de la vk plasmatique comme marqueur?

A
VK plasmatique (reflete les apports récents)
etr
How well did you know this?
1
Not at all
2
3
4
5
Perfectly
25
Q

Comment le PIVKA prothrombine ou Ostéocalcine sous carboxylée. (PIVKA2) permet de voir la carence en vit K?

A

on mesure les formes sous carboxylée. cest la forme porteuse du residu GLU, qui est avant l’action de la carboxylase, quand on a une carence on a un proble des GLU en GLA’ et on a lAccumulation de prothrobmine ou dostehocalcyne, si ça s’accumule c’Est indicateur d’une carence en vit K.

How well did you know this?
1
Not at all
2
3
4
5
Perfectly
26
Q

Décrivez les ANREF pour la vit K?

Quel indicateur a été choisi et pourquoi?

Quelle est l’exception?

Quel est le nombre par jour pour les adultes?

A

Indicateur spécifique:

la vit K est un Apport suffisant.
Apports médians de la population américaine
pour chaque groupe d’âge avec ces apports medians, on a aucun signe de carence, on conclu donc que ces apports sont suffisants.

( sauf pour nourisson, vient du contenu en lait maternel, pas élevé mais serait suffisant. )

Adultes: 90-120 ug

How well did you know this?
1
Not at all
2
3
4
5
Perfectly
27
Q

Décrivex l’AMT pour la vit K.

A

Aucune toxicité documentée: Pas d’AMT

How well did you know this?
1
Not at all
2
3
4
5
Perfectly
28
Q

Une personne carencée en vitamine K peut…
Plusieurs réponses sont possibles.

Démontrer des signes de coagulation sanguine perturbée tels la présence d’un nombre anormalement élevé de caillots sanguins.

Être sous thérapie à la Warfarine depuis longtemps

Souffrir de cholestase

Avoir un temps de prothrombine augmenté

Avoir des niveaux sanguins abaissés d’ostéocalcine sous-carboxylée.

A

Une personne carencée en vitamine K peut…
Plusieurs réponses sont possibles.

Démontrer des signes de coagulation sanguine perturbée tels la présence d’un nombre anormalement élevé de caillots sanguins.
Mauvaise réponse.
Il est vrai que la coagulation sanguine sera perturbée mais celle-ci sera diminuée, ce qui mènera plutôt aux hémorragies et non à la formation de caillots.

Être sous thérapie à la Warfarine depuis longtemps
Bonne réponse.
La Warfarine est un médicament anti-coagulant qui inhibe les enzymes permettant le recyclage de la vitamine dans le cycle de la vitamine K. Lors d’un traitement à la Warfarine, le corps devient très dépendant des sources exogènes de vitamine K afin de lui permettre d’avoir des quantités suffisates de vitamine K pour assurer ses fonctions.

Souffrir de cholestase
Bonne réponse.
La cholestase est une condition caractérisée par un défaut de sécrétion biliaire. La bile étant nécessaire à la formation des sels biliaires requis pour l’émulsification des lipides et vitamines liposolubles (dont la vitamine K), un défaut de sa sécrétion aura pour conséquence de mener à la malabsorption de ces nutriments.

Avoir un temps de prothrombine augmenté
Bonne réponse.
Une carence en vitamine K mène à une dysfonction des protéines impliquées dans la coagulation sanguine. Celles-ci n’assureront plus une coagulation sanguine normale et le temps de prothrombine augmentera considérablement.

Avoir des niveaux sanguins abaissés d’ostéocalcine sous-carboxylée.
Mauvaise réponse.
Non-mesuré cliniquement, l’ostéocalcine sous-carboxylée est un marqueur du statut nutritionnel en vitamine K utilisé en recherche. La carence en vitamine K aura pour effet de nuire à la carboxylation des protéines porteuses d’un résidu d’acide GLU. Il y aura donc accumulation de ces protéines sous-carboxylées en circulation sanguine.

How well did you know this?
1
Not at all
2
3
4
5
Perfectly
29
Q

Choisissez l’affirmation exacte.

Les phyolloquinones sont particulièrement abondants dans les aliments fermentés

Le foie est l’organe du corps où est particulièrement concentré la vitamine K.

La vitamine K circule dans le sang lié à l’albumine

Les protéines vitamine K-dépendantes sont uniquement impliquées dans la coagulation sanguine.

La vitamine K dispose d’un transporteur spécifique sur l’entérocyte pour être absorbé.

A

Le foie est l’organe du corps où est particulièrement concentré la vitamine K.

How well did you know this?
1
Not at all
2
3
4
5
Perfectly
30
Q

Choisissez l’affirmation exacte.

Les femmes enceintes sont particulièrement à risque de développer une carence en vitamine K

L’apport médian de la population américain est l’indicateur ayant servi à établir l’apport suffisant en vitamine K

Comme toutes les vitamines liposolubles, la vitamine K est particulièrement toxique.

L’antibiothérapie n’est pas considérée comme une cause potentielle à une carence en vitamine K.

Un temps de prothrombine normal est l’indicateur ayant servi à établir l’apport suffisant en vitamine K

A

L’apport médian de la population américain est l’indicateur ayant servi à établir l’apport suffisant en vitamine K

How well did you know this?
1
Not at all
2
3
4
5
Perfectly
31
Q

Quelles sont les 4 formes de vitamine A dans le règne animal?

Dans

A

rétinal

rétinol vit A préformée

acide rétinoique forme oxydée

esters de rétinol

How well did you know this?
1
Not at all
2
3
4
5
Perfectly
32
Q

Quellesest la forme de vitamine A dans le règne vegetal?

A

des provitamines A

How well did you know this?
1
Not at all
2
3
4
5
Perfectly
33
Q

Nommez des exemples de provitamines A

A
caroténoides
alpha carotene
beta carotene
gamma carotene
cryptoxanthine
How well did you know this?
1
Not at all
2
3
4
5
Perfectly
34
Q

Pourquoi fruits et légumes verts ont aussi des carotenoide?

A

Le vert vient de la chlorophylle et masque le pigment typique des carotenoides plus orangé.

How well did you know this?
1
Not at all
2
3
4
5
Perfectly
35
Q

Quelles sont les 4 formes de vitamine E?

A

Alpha, beta, gamma, delta

How well did you know this?
1
Not at all
2
3
4
5
Perfectly
36
Q

Des formes Alpha, beta, gamma, delta, de vitamine E, quelles sont les proportions respectives dans notre alimentation?

A

Alpha = 75 - 80%, gamma = 15-20%

How well did you know this?
1
Not at all
2
3
4
5
Perfectly
37
Q

Il existe plusieurs formes de alpha vitamine E, qu’est-ce qui distingue ces formes?

A

Ce sont différents stéréoisomères.

How well did you know this?
1
Not at all
2
3
4
5
Perfectly
38
Q

Quel est le stereoisomere de la alpha vitamine E le plus frequent dans l’alimentation?

A

RRR-alpha tocopherol

How well did you know this?
1
Not at all
2
3
4
5
Perfectly
39
Q

Dans les supplements, quels sont les stereoisomeres de alpha tocopherol que l’on met?

A quelle autre molecules doit on complexer la vitamine E? pourquoi?

A

Dans suppléments:
All-rac-α tocopherol (tous les stereoisomeres - on ne sait pas pourquoi)

Esters de tocophérol
(succinate ou acétate)
Dans les supplements, le tocopherol doit etre complexé, pour le stabiliser.

How well did you know this?
1
Not at all
2
3
4
5
Perfectly
40
Q

Vrai ou faux sur les sources:

1. Les aliments ne contiennent pas de rétinol libre

A

Vrai, le retinol libre n’est pas reconnu pour etre present dans les aliments, dans les aliments il est lié a un esther, souvent un acide gras.

How well did you know this?
1
Not at all
2
3
4
5
Perfectly
41
Q

Vrai ou faux sur les sources:

2. Il n’y a pas de caroténoïdes dans les aliments d’origine animale

A

Faux, il y a des carotenoides animaux, dans les viandes, voilailles, poisson, oeufs, Il en a beaucoup dans le jaune d’oeuf, (la lutéine)

How well did you know this?
1
Not at all
2
3
4
5
Perfectly
42
Q

Vrai ou faux sur les sources:

3. La vitamine E n’est présente que dans le règne végétal

A

Vrai, s’il y en a dans le regne animal, il n’y a aucune source qui ressort de façon marquée. Dans certaines sources animales se seraient ajoutés ou des traces. Ce n’est pas ce qui compte dans les apports en vitamine E.

On a de la vit E dans les fruits et legumes malgre le faible contenu en gras, pourquoi? Car les FL ont aussi des membranes a base de phospholipides et la vit E s’inserent dans les membranes.

How well did you know this?
1
Not at all
2
3
4
5
Perfectly
43
Q

Vrai ou faux sur les sources:

  1. La vitamine E n’est présente que dans les aliments riches en lipides
A

Faux, On a de la vit E dans les fruits et legumes malgre le faible contenu en gras, pourquoi? Car les FL ont aussi des membranes a base de phospholipides et la vit E s’inserent dans les membranes.

How well did you know this?
1
Not at all
2
3
4
5
Perfectly
44
Q

COmment se fait la digestion de la vit A, E et des carotenoides?

A
  1. Incorporation dans micelles
  2. Action enzymatique
    Pancréas: Lipase (esters de rétinol) et Estérase (esters de tocophérol)
    donc la lipase libere les AG qui sont liés au retinol
    • Bordure en brosse: Retinyl ester hydrolase (esters de rétinol) hydrolyser l’ester de retinol pour separer l,AG de son retinol
    •meme chose pour la lipase pui l’esterase pour le tocopherol.

EN résumé, la lipase agit sur la vit A et la vit E, la retinyl ester hydrolase agit sur la vit A seulement.

LEs 2 enzymes retirent l’AG du retinol ou du tocopherol respectivement.

How well did you know this?
1
Not at all
2
3
4
5
Perfectly
45
Q

Décrivez comment se fait l’absoprtion de la vit A, E et des carotenoides?

A

Absorption

  • Rétinol : Transporteur : taux absorption: 80%
  • Caroténoïdes: Voie passive: 5-20%
  • Vitamine E : Transporteur SR-B1 : taux absorption: >50%
How well did you know this?
1
Not at all
2
3
4
5
Perfectly
46
Q

Que se passe t’il avec le retinol une fois absorbé par l’enterocyte?

A

Re esterifier puis envoyé au chylomichron

ou bien métabolisé en retinal puis en acide retinoique qui diffuse dans le sang

How well did you know this?
1
Not at all
2
3
4
5
Perfectly
47
Q

Que se passe t’il avec les carotenoides une fois absorbé par l’enterocyte?

A

(transf. caroténoïdes en rétinol)

(pas uniquement dans entérocyte qu’on a la capacité de bioconversion des caroténoïdes; au niveau des poumons aussi sa se fait)

• Incorporation chylo

How well did you know this?
1
Not at all
2
3
4
5
Perfectly
48
Q

Que se passe t’il avec la vitamine E une fois absorbé par l’enterocyte?

A

Vitamine E
Incorporation
chylomicrons ou HDL
HDL qui vient de l’enterocyte est une voie secondaire.

How well did you know this?
1
Not at all
2
3
4
5
Perfectly
49
Q

La biodisponibilté des carotenoides est de 5-20%, quels facteurs l’augmentent ou la diminue? (4 facteurs)
Indices:

A
  • Présence de lipides (ingestion sumultanée de lipides)
  • Degré de cuisson (ramollit matrice alimentaire)
  • Matrice alimentaire (supplément versus aliment)
  • Compétition intestinale (caroténoïde sont une famille mangé ensemble; compétition entre eux); Présence autres composantes alimentaires (fibres, autres caroténoïdes non-provitamine A)
How well did you know this?
1
Not at all
2
3
4
5
Perfectly
50
Q

Décrivez l’importance du foie pour la vitamine A et E. (long développement)

Indices:

cellules stellaires

Excretion

transport

utilisation ou entreposage.

A

Les cellules stellaires du foie sont un site de réserve de 70% du retinol du corps, elles sont mobilisées si le corps en a besoin.

Les hepatocytes vont excréter les vit liposolubles en trop via la bile et les celles, generalapement pas réabsorbé dans la circulation entérohépatique.

Ils font aussi les protéines sériques de transport du rétinol et la transthyrétine, qui permettent de la transporter.

Ils font aussi l’albumine aui aide au transport de l’acide rétinoique

Ils font aussi les lipoprotéines qui permettent de transporter les carotenoides, les esters de retinol et la vitamine E. Mais c’est secondaire.

Ces mecanismes de transport permettent damener la vit A et E au tissus cibles ou pour les carotenoides et vitamine E detre entreposé dans le tissu adipeux.

How well did you know this?
1
Not at all
2
3
4
5
Perfectly
51
Q

Comment la vitamine E est elle transportée dans l’organisme? quelle forme de la vit E est surtout transportéE?

A

Via la alpha-TTP (alpha tocopherol transport protein), qui va se lier au VLDL pour aller en circulation.

α-T = 100%
γ-T = 9%
How well did you know this?
1
Not at all
2
3
4
5
Perfectly
52
Q

Décrivez le mécanisme de la vitamine E dans son role antioxydant. (long developpement)

A

La vit E est incorporée aux membranes, lorsque un phospholipides est oxydée par un radical libre ou un peroxyde, ça crée une reaction en chaine, eventuellement le tocopherol va accepter un electron et va etre sous forme radical tocopheroxyl. En presence de bons niveaux de vitamine C, le tocopheroxyl oxyde la vit C et est recyclé en tocopherol. La vit C est maintenant sous forme ascorbyl peroxyde, cette réaction est arretée par le glutathion, qui recycle la vit C.

How well did you know this?
1
Not at all
2
3
4
5
Perfectly
53
Q

Pourquoi en carence de vit C la vit E devient pro oxydante?

A

Elle ne peut est recyclée

How well did you know this?
1
Not at all
2
3
4
5
Perfectly
54
Q

Sur quelle partie de l’oeil l’acide retinoique et le retinal agissent respectivement?

A

La cornée et la rétine.

How well did you know this?
1
Not at all
2
3
4
5
Perfectly
55
Q

Sur quel cellules le retinal agit il specifiquement? Quel est le role de ces cellules?

A

Les batonnets photorecepteurs (cônes (lumière vive) et bâtonnets (lumière restreint; donc importance du rétinal)) transmet information au cerveau

How well did you know this?
1
Not at all
2
3
4
5
Perfectly
56
Q

Décrivez le metabolisme du retinol dans l’oeil et comment cela est lié aux signaux de vision envoyé au cerveau.

A

Le retinol arrive dans la cellule epitheliale retinienne via le RBP,

il est metabolisé en retinal.

Celui ci est transporté au batonnet et permet le recyclage du rhodopsin.

Lorsqu’un photon interragit avec le rhodoppsin, il devient opsin + retinal, et cette reaction cause un signal qui transmet de l’info au cerveau.

Le retinal est remetabolisé en retinol puis retourne a la cellule epitheliale retinienne.

How well did you know this?
1
Not at all
2
3
4
5
Perfectly
57
Q

Qu’est-ce qui fait que la carence en Vit A se manifeste tres tardivement dans l’oeil?

A

Le retinol est recyclé, dans le métabolisme de l,oeil, alors c’est long avant que l’on en manque.

How well did you know this?
1
Not at all
2
3
4
5
Perfectly
58
Q

Quels sont les autres tissus cibles de la vitamine A? Sous quelle forme fait elle ces autres fonctions? (3 fonctions et 1 autre forme)

A

Epithelia (peau, cornée):
acide rétinoïque

Embryon: acide rétinoïque

Cellules système immunitaire:
acide rétinoïque

How well did you know this?
1
Not at all
2
3
4
5
Perfectly
59
Q

Quel est le role general de l’acide retinoique?

A

Hormone agissant sur le noyau

How well did you know this?
1
Not at all
2
3
4
5
Perfectly
60
Q

Décrivez le mecanisme cellulaire par lequel l’acide retinoique est une hormone. (long developpement)

A

Un heterodimere RXR et RAR est présent dans le noyau.

D’un coté le retinol est métabolisé en une forme d’acide retinoique qui se lie au coté RAR de l,heterodimere.

D,une autre coté, l’acide retinoique circulant se lie au coté RXR de l’heterodimere.

Ce complexe va reconnaitre la sequence RARE de l’ADN et va influencer sa transcription, ce qui aura une influence sur la synthese de proteine en augmentant ou diminuant dependamment du gene.

How well did you know this?
1
Not at all
2
3
4
5
Perfectly
61
Q

Nommez un mecanisme très important pour le développement auquel l’acide retinoique participe grace a son action nucleaire.

En quoi cela va influencer la peau?

Le développement de l’embryon?

A

Différenciation cellulaire.

la peau a des cellules basales et l’acide retinoique va permettre la différenciation en cellule keratinocyte plus specialisé

L’Embryon est un tas de cellule souche, sans acide retinoique ou avec trop d’acide retinoique, ça va causer de sproblemes de développement ou malformation.

How well did you know this?
1
Not at all
2
3
4
5
Perfectly
62
Q

L’industrie pharmaceutique utilise la propriété de différenciation cellulaire de l’acide retinoique, pour quelles applications et quels sont les effets secondaires possibles?

Quelle forme est utilisée?

QUelles utilisations (4)

Effets secondaires possibles ? (4)

A

13-trans acide rétinoïque ou un mélange
de rétinoïdes synthétiques

Acné (accutane), psoriasis, Anti-rides,
contre les taches de pigmentation

3) Peut causer des signes de toxicité légère
(surtout Accutane): chéilite,
xérostomie, conjonctivite, douleurs
musculosquelettiques

How well did you know this?
1
Not at all
2
3
4
5
Perfectly
63
Q

Décrivez l’Action de l’Acide retinoique au niveau de l’embryon

A

VA permettre la différenciation du tissu neural anterior pour la neurogenese et le “anterior posterior patterining” et permettra le développement du systeme cardiovasculaire et musculosquelletique.

How well did you know this?
1
Not at all
2
3
4
5
Perfectly
64
Q

Plusieurs causes de carence sont possibles pour la vitamine A et E, décrivez celles en lien avec les apports.

A

Surtout dans les pays en developpement, on manque d’apport, ça cause surtout la deficience en vit A”

How well did you know this?
1
Not at all
2
3
4
5
Perfectly
65
Q

Plusieurs causes de carence sont possibles pour la vitamine A et E, décrivez celles en lien avec la digestion. (3 principales)

A

Cause la carence en vit A et E

Fibrose kystique

Cholestase
Maladies hépatiques:
Cirrhose, hépatite…
(car elles touchent les voies biliaires)

Pancréatite
manque d’enzymes.

How well did you know this?
1
Not at all
2
3
4
5
Perfectly
66
Q

Plusieurs causes de carence sont possibles pour la vitamine A et E, décrivez celles en lien avec la l’absoprtion. (3 principales)

A

Cause la carence en vit A et E

Hypobetalipoprotéinémie
(pas d’apoB)

Abeta lipoprotéinéinémie
(pas de MTP) (MTP= protéine permettant de lipidé les lipoprotéine (donc apoB dans intestin et apo-II dans foie))

Syndrome intestin court (atrophie de villosités intestinale = diminution surface absorption)
Maladie coeliaque
MII (maladies inflammatoires de l’intestin (exemple crohn disease))

How well did you know this?
1
Not at all
2
3
4
5
Perfectly
67
Q

Plusieurs causes de carence sont possibles pour la vitamine A et E, décrivez celles en lien avec le Foie. (3 principales)

A

Cause la carence en vit A

Alcoolisme
Substances
hépatotoxiques (exemple acetaminophene (tylenol))

How well did you know this?
1
Not at all
2
3
4
5
Perfectly
68
Q

Plusieurs causes de carence sont possibles pour la vitamine A et E, décrivez celles en lien avec le Transport. (3 principales)

A

Cause la carence en vit E

Absence alpha-TTP

How well did you know this?
1
Not at all
2
3
4
5
Perfectly
69
Q

Comment évalue on le statut en vitamine E? (3 facons principales), chaque facon principale inclut quelques indicateurs (2, 2 ou 4)

A

Biochimiques
α-tocophérol sérique
α-tocophérol Tissu adipeux

Fonctionnels
Hémolyse globules rouges
Péroxydation lipidique

Cliniques Physiques
DYSTROPHIE AXONALE
• Perte des réflexes
• Altération perceptions (vibratoires et propioceptives)
• Ataxie: problème coordination mouvements
• Ophtalmoplégie

How well did you know this?
1
Not at all
2
3
4
5
Perfectly
70
Q

Évaluation statut nutritionnel en vitamine E et signes de
carence

Quels sont les domaines d’application de :
α-tocophérol sérique
α-tocophérol Tissu adipeux

A

α-tocophérol sérique : clinique

α-tocophérol Tissu adipeux : recherche

How well did you know this?
1
Not at all
2
3
4
5
Perfectly
71
Q

Évaluation statut nutritionnel en vitamine E et signes de carence

Pourquoi l’Hémolyse globules rouges
et la Péroxydation lipidique indique une carence?

A

hemolyse est un signe de stress oxydant, quand ils sont stressés ils vont hémolyser facilement.
péroxydation lipidique:
peut etre dans l’urine ou le sang. si ça augmente ça peut indiquer que le statut en vit E est plus faible.

How well did you know this?
1
Not at all
2
3
4
5
Perfectly
72
Q

Comment reconnait on l’ophtalmoplégie? Qu’Est-ce que cela indique en terme d’apports en nutriment?

A

yeux qui louche (carbon); réversible et lié à neurotransmission

Carence en vitamine E

How well did you know this?
1
Not at all
2
3
4
5
Perfectly
73
Q

QUels sont les ANREF pour la vitamine E? Comment ont ils été fixés (quel indicateur)

A

BME: 12 mg
ANR: 15 mg

BME = Quantité de vit.E pour
prévenir l’hémolyse des globules
rouges chez 50% des sujets

How well did you know this?
1
Not at all
2
3
4
5
Perfectly
74
Q

Pour la vit E:

BME = Quantité de vit.E pour
prévenir l’hémolyse des globules
rouges chez 50% des sujets

QUelle réaction utilise on pour vérifier cela?

A

on vérifie si les globules rouges en presence de H2O2 vont lyser ou non.

How well did you know this?
1
Not at all
2
3
4
5
Perfectly
75
Q

Ya t’il des recommandations particulières pour les apports en vit E? (ex: plus de vit E pour certains groupes?)

A

Recommandations
MCV Exercice Tabagisme
Ces conditions sont associées à une augmentation
du stress oxydatif mais il n’existe aucune
recommandation particulière pour la vitamine E

How well did you know this?
1
Not at all
2
3
4
5
Perfectly
76
Q

Quel est l’AMT pour la vit E? Quel était le critere retenu pour l’établir?

A
AMT
1000 mg PAR JOUR
SOMME DE TOUTES LES FORMES
DE VITAMINE E (ALIMENTAIRE + SUPPLÉMENTS)
Critère retenu: Hémorragies
How well did you know this?
1
Not at all
2
3
4
5
Perfectly
77
Q
Vit E: AMT
1000 mg PAR JOUR
SOMME DE TOUTES LES FORMES
DE VITAMINE E (ALIMENTAIRE + SUPPLÉMENTS)
Critère retenu: Hémorragies

Quel mécanisme serait responsable des hémorragies?

A

problème de coagulation sanguine; serait un problème d’intéraction négative entre vitamine E et vitamine K (trop de vitamine E nuirait à vitamine K qui est pro-coagulante)

How well did you know this?
1
Not at all
2
3
4
5
Perfectly
78
Q

quels sont les Marqueurs de l’état nutritionnel en vitamine A? (2 grandes catégories, 5 sous catégories au total)

Décrivez les avantages ou inconvénients et l’utilisation de chaque marquer.

A

Biochimiques:

Rétinol plasmatique: n’est pas un bon marqueur mais plus utilisé

Rétinol/acide rétinoïque urinaire: métabolites inactifs

Rétinol lait maternel: reflète réserves mère (Contexte expérimental

Rétinol hépatique: invasif (meilleur marqueur mais biopsie donc moins utilisé)

Cliniques physiques:
diminution Adaptation vision nocturne Cécité crépusculaire ou héméralopie:
Signe carence avancée

How well did you know this?
1
Not at all
2
3
4
5
Perfectly
79
Q

Pourquoi le rétinol plasmatique n’est pas un bon marqueur?

A

pas un marqueur sensible donc indique juste quand c’Est grave.

How well did you know this?
1
Not at all
2
3
4
5
Perfectly
80
Q

Décrivez les 5 contextes dans lequel le retinol plasmatique ne restera pas tres consant:

A

Il reste tres content sauf
I- Épuisement réserves hépatiques
II- Défaillance hépatique ((très important car RBP et transtyrétine produit par le foie permet transport du rétinol en circulation)
III- Déficience en zinc
IV- Malnutrition protéino-énergétique ((pas de synthèse protéique de protéine de transport de rétinol)᳀)
V- Inflammation, infection, trauma (Causera une baisse de la RBP)

How well did you know this?
1
Not at all
2
3
4
5
Perfectly
81
Q

POURQUOI LE ZINC EST SI IMPORTANT POUR LE

MÉTABOLISME DU RÉTINOL? (2 raisons)

A
1- Entre dans la composition de la RBP
Carence en zinc: ß synthèse RBP:
diminue la libération du rétinol du foie
2- Co-facteur de l’enzyme qui convertit le rétinol
en rétinal
How well did you know this?
1
Not at all
2
3
4
5
Perfectly
82
Q

Décrivez les principaux symptomes de carence de la vitamine A. (4 grandes catégories de symptomes, quelques sous catégorie dépendamment de la catégorie)

A

Rôle vision

  Adaptation vision nocturne lié au manque de rétinal

  Syndrome xérophtalmique lié au manque d’acide rétinoïque (ici peut atteindre un stade irréversible contrairement à adaptation nocturne)

Systeme immunitaire

  Susceptibilité accrue aux infections

Morphogenèse

  Tératogenèse (malformation du foetus)

Différenciation

   Hyperkératose folliculaire sécrétion anormalement 
   élevé de kératine
How well did you know this?
1
Not at all
2
3
4
5
Perfectly
83
Q

Décrivez la cascade d’événement du syndrome xérophtalmique et a partir de quel symptome il est irréversible. (long développement)

A

Commence par une secheresse des yeux, puis on craint la lumiere vive, les paupieres vont etre inflammé ensuite, l’epithelium de la cornée épaissie, puis on aura la xérose qui est une sécheresse généralisée

Frontière Irréversible : tache de bitot (tache blanche dans l’oeil (soit un dépot de kératine)

Kératomalacie
cornée blancheatre,
Perforation de la cornée
Cécité

How well did you know this?
1
Not at all
2
3
4
5
Perfectly
84
Q

Une déficience en vitamine A cause l’hyperkératose folliculaire, décrivez ce problème et pourquoi il se produit.

A

Cellules sécrétrices mucus (peau)
La déficience en vit A fait que ces cellules deviennent des cellules sécrétrices de kératine due a un probleme de différenciation.

Cela fait épaissir la cornée et cause l’hyperplasie de ces cellules sécrétrices de kératine

How well did you know this?
1
Not at all
2
3
4
5
Perfectly
85
Q

Carence en vit A: La kératinisation des épithélia va affecter d’autres tissus que la peau, nommez les 3 principaux et les conséquences

A
  • Voies respiratoires: augmente infections
  • Oeil: conjonctive et cornée: Taches de Bitot, baisse de l’acuité visuelle mene à ulcération, perforation
  • Voies urinaires, Vessie: prédisposition calculs urinaires
How well did you know this?
1
Not at all
2
3
4
5
Perfectly
86
Q

Quels sont les ANREF pour la vit A?

Quel critere a été utilisé pour les établir?

A
BME ANR
H: BME: 625 ug ÉAR
ANR: 900 ug ÉAR
F: BME: 500 ug ÉAR
ANR: 700 ug ÉAR

Critère: Maintien de réserves hépatiques suffisantes de vitamine A
pour faire face aux besoins accrus (stress, maladies, faible apport)

How well did you know this?
1
Not at all
2
3
4
5
Perfectly
87
Q

Placer les chiffres suivants dans les pharases ci-dessous

24, 12, 1, 2

Rétinol alimentaire, _ μg donne 1μg rétinol
=1EAR

Autres caroténoïdes: \_\_ μg 
Alpha-carotène
Gamma-carotène
Cryptoxanthine 
Donne
1μg rétinol
= 1EAR

β-carotène (suppléments, huile), __μg donne 1μg rétinol
=1EAR

β-carotène (aliments, végétaux), __ μg donne 1μg rétinol =1EAR

A

Rétinol alimentaire, 1 μg donne 1μg rétinol
=1EAR

Autres caroténoïdes: 24  μg 
Alpha-carotène
Gamma-carotène
Cryptoxanthine 
Donne
1μg rétinol
= 1EAR

β-carotène (suppléments, huile), 2 μg donne 1μg rétinol
=1EAR

β-carotène (aliments, végétaux), 12 μg donne 1μg rétinol =1EAR

How well did you know this?
1
Not at all
2
3
4
5
Perfectly
88
Q
ANREF-Caroténoides
\_\_\_\_\_\_\_\_ recommandation
quant à un % de vitamine A
devant provenir des provitamines A
5 portions Fruits et légumes:
5-6 mg de provitamine A
soit environ la \_\_\_\_\_\_ 
des recommandations de vitamine A
A
ANREF-Caroténoides
Aucune recommandation
quant à un % de vitamine A
devant provenir des provitamines A
5 portions Fruits et légumes:
5-6 mg de provitamine A
soit environ la moitié
des recommandations de vitamine A
How well did you know this?
1
Not at all
2
3
4
5
Perfectly
89
Q

Toxicité (vitamine A)

La vitamine A est l’une des vitamines les ______ toxiques

A

Toxicité (vitamine A)

La vitamine A est l’une des vitamines les plus toxiques

How well did you know this?
1
Not at all
2
3
4
5
Perfectly
90
Q

Toxicité (vitamine A)

Quelles sont les causes principales de toxicités?

A
  • Excès de suppléments (rétinol)
  • Pratiques alimentaires particulières (+++ consommation de foie)
  • Prise de certains médicaments (13-cis-RA): Accutane
How well did you know this?
1
Not at all
2
3
4
5
Perfectly
91
Q

Toxicité (vitamine A)

Quelles sont les principales conséquences? (2 consequences)

A

Déstabilise
les membranes
Cause douleurs osseuses
et articulaires

Cause expression
inappropriée
de certains gènes
tératogène

How well did you know this?
1
Not at all
2
3
4
5
Perfectly
92
Q

Toxicité (vitamine A)
Quelles sont les principales conséquences?
Donnez un exemple de conséquences au niveau tératogène

A

fissure palatine (bec de lièvre) lié à excès vitamine A chez la mère enceinte

How well did you know this?
1
Not at all
2
3
4
5
Perfectly
93
Q

Toxicité du rétinol
AMT
Parle on d’ÉAR ou bien de retinol?

A

ICI AMT ON PARLE DE RÉTINOL DIRECTEMENT PAS D’ÉQUIVALENT DE RÉTINOL, CAR ÉQUIVALENT SONT PAS TOXIQUE

How well did you know this?
1
Not at all
2
3
4
5
Perfectly
94
Q

Toxicité du rétinol
AMT
Quel est il?

A

3000 ug PAR JOUR

Rétinol

How well did you know this?
1
Not at all
2
3
4
5
Perfectly
95
Q

Toxicité du rétinol
AMT

Selon quel facteur les criteres retenus pour fixer l’AMT varie il?

A

Critères retenus varient selon l’âge:

How well did you know this?
1
Not at all
2
3
4
5
Perfectly
96
Q

Toxicité du rétinol
AMT
Critères retenus varient selon l’âge:
Quel est le critère pour:

Les nourrissons

Les hommes et femmes de plus de 19 ans

Les femmes enceinte

A
  • Nourrissons: augmentation pression crânienne
  • Hommes et femmes >19 ans: Troubles hépatiques

-Grossesse (19-50 ans): Tératogenèse
** Attention femmes qui suivent un
Traitement Accutane

How well did you know this?
1
Not at all
2
3
4
5
Perfectly
97
Q

Vrai ou faux, il n’y a pas d’AMT pour les carotenoides

A

Pas d’AMT

How well did you know this?
1
Not at all
2
3
4
5
Perfectly
98
Q

Quel groupe de la population doit faire attention aux suppléments en caroténoides? PourquoI?

A

Liens possibles entre la prise d’un supplément de
Bêta-carotène (20 mg et +) et le cancer du poumon chez
les fumeurs.

How well did you know this?
1
Not at all
2
3
4
5
Perfectly
99
Q

Quelle est la recommandation pour la population générale en lien avec les supplements de beta carotene?

Est-ce que ça s’applique aussi aux aliments?

A

Il est déconseillé pour la population générale de
consommer des suppléments de b-carotène
en absence de déficience en vitamine A.
(cette recommandation particulière ne s’applique
pas aux aliments)/

How well did you know this?
1
Not at all
2
3
4
5
Perfectly
100
Q

Dans quel cas les carotenoides peuvent etre toxique? QUelle est la conséquence?

A

Caroténodermie:

ingestion supplément bêta carotène: >30 mg/j

How well did you know this?
1
Not at all
2
3
4
5
Perfectly
101
Q

L’AMT de la vit A est fixé a 3000 ug par jour, mais quel risque augmente a partir de 1500 ug par jour? Pourquoi?

A
ASSOCIÉ A UN
ACCROISSEMENT DES FRACTURES
OSTÉOPOROTIQUES ET DIMINUTION
DENSITÉ MINÉRALE OSSEUSE
RÉTINOL OU VITAMINE A PRÉFORMÉE
EST ICI EN CAUSE
How well did you know this?
1
Not at all
2
3
4
5
Perfectly
102
Q

Que sont les vitamines?

A

vitamines sont un groupe de molecule enzymatique qui sont cofacteurs (pour la plupart)

How well did you know this?
1
Not at all
2
3
4
5
Perfectly
103
Q

Vrai ou faux, la choline est un cofacteur de reaction enzymatique

A

Choline: pas un cofacteur,

ex: phosphatidyl choline fait partie des membranes.

How well did you know this?
1
Not at all
2
3
4
5
Perfectly
104
Q

Quelle est la différence entre les folates et folacine
et
L’acide folique?
(indice: sémantique)

A

Folates et Folacine: Termes génériques
Noms qui englobent toutes les formes
Acide folique: Nom donné à la forme synthétique (celle
retrouvée dans les suppléments et aliments fortifiés)

How well did you know this?
1
Not at all
2
3
4
5
Perfectly
105
Q

Quels sont les groupements chimiques qui font toujours partie de la molecule de folate? Lesqules varient et distingue les formes?

A

folate a toujours la pteridine et para

ce qui distingue sont les résidus glutamate. chaine tres courte ou tres longue.

How well did you know this?
1
Not at all
2
3
4
5
Perfectly
106
Q

Qu’est ce qui determine le taux d’Absorption des folates?

A

plus la chaine de residus glutamates est longue plus cest difficile a absorber.

quand il y en a plusieurs, ce sont des polyglutamates, il y a des etapes de plus dans lintestin donc ca reduit la biodisp

How well did you know this?
1
Not at all
2
3
4
5
Perfectly
107
Q

FOLATE:

Faites les bonnes associations:

Résidus monoglutamate
Résidus Polyglutamate

Regne vegetal
regne animal

A

monoglutamates sont de 1 -3 souvent le folates du residu animal + l’acide folique

souvent les folates des sources vegetales sont plus des polyglutamates, donc moins bonne biodisponibilité

How well did you know this?
1
Not at all
2
3
4
5
Perfectly
108
Q

Quelle est la molecule active de folate dans le corps? Qu’est-ce qui la distringue?

A

Tetrahydrofolate

Ions H sur la pteridine et Para

How well did you know this?
1
Not at all
2
3
4
5
Perfectly
109
Q

Vrai ou faux, la tetrahydrofolate peut seulement etre métabolisée par le corps.

A

Faux Elle peut etre soit mangée sous cette forme, ou elle peut etre converti sous cette forme dans le corps.

How well did you know this?
1
Not at all
2
3
4
5
Perfectly
110
Q

Quel groupement peut etre ajouté a la molecule de folate pour faire varier son action?

A

Segment monocarboné.

How well did you know this?
1
Not at all
2
3
4
5
Perfectly
111
Q

Folate

Vrai ou faux, le segment carboné méthyle vient seulement du métabolisme dans le corps.

A

peut attacher des fragments monocarbonés comme le methyl. cest une forme methylée alimentaire mais le corps peut aussi la méthylée.

How well did you know this?
1
Not at all
2
3
4
5
Perfectly
112
Q

Folate

Vrai ou faux, les autres segments monocarbonés que le methyle peuvent etre consommé dans l’alimentation.

A

Faux

les autres formes de segments monocarbonés peuvent seulement etre ajoutés dans le corps.

How well did you know this?
1
Not at all
2
3
4
5
Perfectly
113
Q

Folate:

Forme alimentaire « naturelle »:
- \_\_\_\_\_\_\_\_\_ réduite
- Mono ou \_\_\_\_\_\_\_\_\_\_\_ (1 à 11 résidus)
- Méthylée (ou non)
- Souvent liée à des \_\_\_\_\_\_\_\_\_\_
Forme synthétique:
- Non-\_\_\_\_\_\_
- \_\_\_\_\_\_\_\_\_(1 résidu)
- Non-\_\_\_\_\_\_\_\_
- Pas bioactive, est activée dans l’organisme
A
Forme alimentaire « naturelle »:
- Habituellement réduite
- Mono ou polyglutamates (1 à 11 résidus)
- Méthylée (ou non)
- Souvent liée à des protéines
Forme synthétique:
- Non-réduite
- Monoglutamate (1 résidu)
- Non-méthylée
- Pas bioactive, est activée dans l’organisme
How well did you know this?
1
Not at all
2
3
4
5
Perfectly
114
Q

Donnez la biodisponibilité des 3 formes de folates:

A

Forme alimentaire « naturelle »: 50%
Forme synthétique (aliments enrichis) 85%
Forme synthétique: (suppléments) 100%

How well did you know this?
1
Not at all
2
3
4
5
Perfectly
115
Q

Pourquoi la biodisponibilité des folates synthetique est différente selon qu’elles soient en supplements ou dans des aliments?

A

dans les aliments (la matrice alimentaire reduit la biodisp)

How well did you know this?
1
Not at all
2
3
4
5
Perfectly
116
Q

Pourquoi utilise-t-on l’acide folique
pour l’enrichissement des aliments et
non d’autres formes de folates?

A

depuis 1998 au canada,

on met l’acide folique car c’est plus stable que les folates. aussi cest rare quon mange des pates cru, riz cru, et le pain est cuit, et l’acide folique resiste a la chaleur et la lumiere, cest la raison pour laquelle on la choisit pour

How well did you know this?
1
Not at all
2
3
4
5
Perfectly
117
Q

Folate (schéma slide 8)

Absorption, Transport, Stockage et Excrétion

Décrivez comment le folate est digéré

A

Enzyme de la bordure en brosse GluγGlutamyl hydrolase transforme les poly en monoglutamate car le PCFT ne reconnait pas les formes qui sont polyglutamates.

How well did you know this?
1
Not at all
2
3
4
5
Perfectly
118
Q

Folate (schéma slide 8)

Absorption, Transport, Stockage et Excrétion

Décrivez comment le folate est absorbé

A

Jéjunum
Transport actif
saturable couplé a des protons. Le transporteur se nomme PCFT

Ce qui n’est pas
absorbé
dans le jéjunum:
va a l’ Iléon et est absorbé passivement.

How well did you know this?
1
Not at all
2
3
4
5
Perfectly
119
Q

Folate (schéma slide 8)

Absorption, Transport, Stockage et Excrétion

Décrivez comment le folate est traité par l’enterocyte.

A

dans lenterocyte, il peut y avoir une methylation qui se fait. Certaines formes de folates vont etre methylée.

How well did you know this?
1
Not at all
2
3
4
5
Perfectly
120
Q

Folate (schéma slide 8)

Absorption, Transport, Stockage et Excrétion

Comment le folate est il transporté? SOus quelle forme est-il nécéssairement a cette etape (indice: residus glutamates)

A

Circule lié
Albumine
dans le sang
(Monoglutamates)

How well did you know this?
1
Not at all
2
3
4
5
Perfectly
121
Q

Folate (schéma slide 8)

Absorption, Transport, Stockage et Excrétion

Ou le folate est il stocké?

A

Moëlle osseuse

Foie (la principale réserve; 1-4 mois)

How well did you know this?
1
Not at all
2
3
4
5
Perfectly
122
Q

Folate (schéma slide 8)

Absorption, Transport, Stockage et Excrétion

Comment le folate est il excrété?

A

Excrétion Fécale

ou via la bile et circulation enterohepatique comme il y en a bcp dans le foie.

How well did you know this?
1
Not at all
2
3
4
5
Perfectly
123
Q

Folate (schéma slide 9)

Rôles

Quel est le role principal des folates.

Ou se produit ce role dans les cellules?

A

Synthese des purines (Bases A et G) dans le cytosol

How well did you know this?
1
Not at all
2
3
4
5
Perfectly
124
Q

Folate (schéma slide 9)

Rôles

Quel est le role secondaire des folates

A

Transférer des segments monocarbonés methyl a d’autres molecules via le cycle methionine homocystéine. Ces segments methyls vont participer a l’epigenetique.

How well did you know this?
1
Not at all
2
3
4
5
Perfectly
125
Q

Quelle autre vitamine du complexe B est essentielle au cycle methionine homocystéine?

A

ici la methylb12 est cofacteur de l’enzyme qui fait la synthese de methionine.

How well did you know this?
1
Not at all
2
3
4
5
Perfectly
126
Q

Quelle autre vitamine du complexe B est essentielle a la synthese des purines ? Via quelle réaction?

A

THF recupere le CH2 dans une reaction de synthese d’AA non essentiels. la vit b6 (pirydoxyne) catalyse l’enzyme de cette réaction donc est un cofacteur.

How well did you know this?
1
Not at all
2
3
4
5
Perfectly
127
Q

Que fait le methylation au niveau de l’ADN?

A

les methyles lient els bases CG. ces bases seront “désastivées” au moment den la traduction et va modifier la synthese de proteines. ce qui peut causer des malfonctions.

How well did you know this?
1
Not at all
2
3
4
5
Perfectly
128
Q

Quelles sont les conséquences d’une hypo ou hyperméthylation de l’ADN?

A
Équilibre des réactions
de méthylation est important
Déséquilibre peut être
pro-cancérigène
Ex: Hyperméthylation gènes de
réparation ADN
\+
Hypométhylation
Pro-oncogènes
How well did you know this?
1
Not at all
2
3
4
5
Perfectly
129
Q

Dans quel cas l’hypermethylation des genes peut etre prooncogene?

Dans quel cas l’hypomethylation des genes peut etre oncogene?

A

quand les deux sont ensembles.

Hyper
on est constamment exposé a des choses qui endommagent l’ADN, il faut avoir des reparation de genes pour palier a ça, donc si c’est hypermethlyé ca n’aiderait pas. prooncogene controle la croissance et la proliferation cellulaire,

Hyperproliferation fait la croissance anormale des cellules qui est a l’origine des cancers.
on met la table pour une apparition de cancer.

How well did you know this?
1
Not at all
2
3
4
5
Perfectly
130
Q

Décrivez l’expérience qui a été conduite sur des souris concernant les folates et l’épignétique

A

Une souris avec une diete normale, une souris avec une diete enrichie de choline, folate, betaine et b12.

On se rend compte que le gene agouti est méthyler chez la souris avec la diete, ce qui a fait qu’elle est mince et brune, alors que l’autre est obese et a plus de chance d’avoir des cancers.

on se demande si ça pourrait aider les humains.

How well did you know this?
1
Not at all
2
3
4
5
Perfectly
131
Q

Marqueurs du statut nutritionnel en folates

Nommez les 3 marqueurs principaux et ce qu’ils refletent.

A
Folates:
globules rouges
Reflète
les réserves:
Diminue
semaines/mois
Après une diminution du 
Folate plasmatique
FOlates:
Sérum
Reflète
l’ingestion
récente; ne corrèle
pas avec les réserves

Homocysteine sérique (si elle s’élève)
Marqueur d’une
déficience
Peu spécifique

How well did you know this?
1
Not at all
2
3
4
5
Perfectly
132
Q

A quel risque pour la santé une homocystéine élevée est elle associée?

A

pas une bonne molecule, on ne veut pas quelle monte dans le sang. quand cest haut cest associé a l’apparition de cancer, et MCV.

How well did you know this?
1
Not at all
2
3
4
5
Perfectly
133
Q

Que sont les ANREF pour les folates?

A

Hommes-Femmes
19-50 ans
BME 320 ug EFA
ANR 400 ug EFA

How well did you know this?
1
Not at all
2
3
4
5
Perfectly
134
Q

Folates:

que sont les EFA?

A

Équivalents folates alimentaires

How well did you know this?
1
Not at all
2
3
4
5
Perfectly
135
Q

Quels sont les 3 indicateurs retenus pour etablir les anref des folates?

A

Niveau normal de folates érythrocytaires
Niveau normal de folates sériques
Niveau normal d’homocystéine sérique

How well did you know this?
1
Not at all
2
3
4
5
Perfectly
136
Q

Folates;

faites les bonnes associations

1 ug ÉFA = 1 ug
1 ug ÉFA = 0,5 ug
1 ug ÉFA = 0,6 ug

acide folique (aliments enrichis)
d’acide folique
folates alimentaires

A

1 ug ÉFA = 1 ug folates alimentaires
1 ug ÉFA = 0,6 ug acide folique (aliments enrichis)
1 ug ÉFA = 0,5 ug d’acide folique

How well did you know this?
1
Not at all
2
3
4
5
Perfectly
137
Q

Y a t’il des recommandations speciales en folates pour certains groupes de la population?

A
ANREF
FEMMES 19-49 ANS
Recommandation spéciale (**non-incluse dans les ANR)
\+ 400μg d’acide folique
(aliments enrichis et/ou suppléments)
How well did you know this?
1
Not at all
2
3
4
5
Perfectly
138
Q

Quel est le moment idéal pour débuter une supplémentation
en acide folique chez la femme en âge de procréer?
a) Dès qu’elle est en âge de tomber enceinte
b) Dès qu’elle commence à planifier une grossesse
c) Dès qu’elle apprend qu’elle est enceinte

A

pourquoi le A prime sur le B? pour tenir compte des accidents de grossesse.

c c’est non, on a besoin de beaucoup de folate pour la fermeture du tube neural, et ça se passe dans le 1er mois de vie, donc generalement on apprend quon est enceinte passé 1 mois, donc est trop tard.

b oui,

mais idéalement des quon est en age de tomber enceinte.

How well did you know this?
1
Not at all
2
3
4
5
Perfectly
139
Q

Quelles sont les 2 raisons pourquoi la déficience en folate représente un défi a diagnostiquer?

A
  1. Anémie mégaloblastique: identique à
    celle induite par la déficience en B12
  2. Déficience en folates survient rarement seule, elle
    est toujours associée à une diète de pauvre qualité,
    alcoolisme, malabsorption
How well did you know this?
1
Not at all
2
3
4
5
Perfectly
140
Q

Comment distingue on la déficience en folate de la déficience en b12?

A

dosage serique

How well did you know this?
1
Not at all
2
3
4
5
Perfectly
141
Q

Schéma du métabolisme des folates

Pourquoi la carence en b12 et en folates induient des symptomes similaires?

A

cest le defaut de synthese de l’ADN qui induit ce type d’anémie.

le manque de b12 induit le defaut d’utilisation des folates. (car le Methyl THF ne peut faire le THF sans la b12”

How well did you know this?
1
Not at all
2
3
4
5
Perfectly
142
Q

Déficience en folate

Décrivez l’anémie mégaloblastique

A

si ADN est de pauvre qualité par manque de folate, l’ADN va etre de pauvre qualité et va se fragmenter facilement. ça fait un megaloblaste (gros globule rouge qui a encore du materiel nucleaire a l’interieur) quand il garde son materiel nucleaire ca donne le signal de contrinuer a grandir .

How well did you know this?
1
Not at all
2
3
4
5
Perfectly
143
Q

folate

Dans le megaloblaste, pourquoi les chromatines se segmentent?

A

car l’ADN est de pauvre qualité car la synthese de bases nucleiques est altérée par le manque de folate.

How well did you know this?
1
Not at all
2
3
4
5
Perfectly
144
Q

Quelle autre cellule du sang peut etre impacté par la déficience en folate? Pourquoi?

A

anemie megaloblastique touche aussi les neutrophiles. manque de folate ca fragmente l’ADN. et la chromatine est segmentée.

How well did you know this?
1
Not at all
2
3
4
5
Perfectly
145
Q

Un individu qui a une anémie mégaloblastique diagnostiquée par
frottis sanguin aura-t-il une hémoglobine et un hématocrite
(proportion de globules rouges dans le volume sanguin) abaissée?

A

les 2 diminues
les megaloblastes ne transportent pas l’Hb efficacement car immatures

aussi le corps a tendance a eliminer les megaloblastes, donc ca diminue le nombre de globule rouge.

How well did you know this?
1
Not at all
2
3
4
5
Perfectly
146
Q

Comment distingue on l’anémie ferriprive de l’anémie megaloblastique? quelle est la ressemblance?

A

dans les deux cas le nombre de globule diminue

le volume de globule va augmenter comparativement a lanemie ferriprive que ça descend.

How well did you know this?
1
Not at all
2
3
4
5
Perfectly
147
Q

Quelle est la caractéristique particuliere des cellules impactées par les carences en folates?

A

toutes les cellules qui se multiplient rapidement vont avoir de la misere (car ont besoin d’ADN). lié a la synthese d’ADN qui ne se fait pas bien

How well did you know this?
1
Not at all
2
3
4
5
Perfectly
148
Q

Quelles sont les cellules qui se renouvellent le plus rapidement dans le corps?

A

quelles sont les cellules qui se renouvelent le plus rapidement? les cellules intestinales, entre 3-5 jours ça tourne.

How well did you know this?
1
Not at all
2
3
4
5
Perfectly
149
Q

Nommez 4 type de tissus spécifique qui vont etre impactés par les carences en folates et les symptomes que cela va causer.

A
  1. Muqueuses: langue, bouche:
    glossite, stomatite angulaire,
    atrophie papillaire
  2. Muqueuse g-i: troubles
    d’absorption, diarrhée
  3. Troubles SNC: irritabilité,
    neuropathie légère
  4. Problèmes de pigmentation:
    peau et ongles
How well did you know this?
1
Not at all
2
3
4
5
Perfectly
150
Q

La carence en folate de la femme peut causer des anomalies du tube neural chez son bebe. DÉcrivez ce qu’est cette anomalie

A

Tube qui donne la moelle epiniere et le cerveau
en se fermant, ca donne lieu a la synthese de moelle epiniere et du cerveau.

ca doit se fermer dans le premier mois de vie.

Si il ne se ferme pas, Selon lendroit ou ça affecte ca peut avoir une multitude d’effet.

How well did you know this?
1
Not at all
2
3
4
5
Perfectly
151
Q

Les bébés qui survivent des anomalies tu tube neural ont generalement une malformation. COmment l’appele on? QUe contient ces malformations?

QUels sont les symptomes?

A

ceux qui survivent ont du spina bifida. Ils survivent avec des sequelles.

Qu’a on dans la poche dans le dos par exemple?
amoncelement de liquide, moelle epiniere, le bebe a des sequelles qui peuvent etre motrice (paraplegie, incontinence ou meme deficit intellectuel).

How well did you know this?
1
Not at all
2
3
4
5
Perfectly
152
Q

Folate

Un adolescent qui a du spina bifida consulte pour perte de poids car il souffre d’obésité. Pourquoi cela semble logique qu,Il souffre d’obésité?

A

histoire de cas, enfant spina bifida qui consultait pour perte de poids, sa motricité était afectée, il consultait pour prise de poids . ça fait du sens considérant lepigenetique que un enfant qui prend du poids aie le spina bifida (possible)

How well did you know this?
1
Not at all
2
3
4
5
Perfectly
153
Q

Vrai ou faux ? Les anomalies du tube neural peuvent survenir malgré
la prise quotidienne d’un supplément de 400
microgrammes d’acide folique.

est-ce que la carence en b12 pourrait faire le meme resultat?

A

vrai, peut survenir,

Ferait du sens mais pas rapporté . donc celle qui prime cest la carence en folate.

How well did you know this?
1
Not at all
2
3
4
5
Perfectly
154
Q

Prévalence d’anomalies du tube neural en fonction des années.

QUe remarque on?

peut on éliminer le risque completement? Pourquoi?

A

ca diminue depuis 1998, l’enrichissement est obligatoire.

ne sera jamais egal a zero, car il y a d’autres causes que la carence en folate.

How well did you know this?
1
Not at all
2
3
4
5
Perfectly
155
Q

Folate

Qu’est-ce que la déficience infra clinique? (insuffisance)?

A

Certaines personnes ont un metabolisme diminué du cycle des folates, ils vont faire plus d’homocysteine, ca va augmenter certaines pathologies.

pas cliniquement visible, mais homocysteine va etre moins converti en methionine,

hausse d’homocysteine peut etre un signe biochimique d’insuffisance en folate.

How well did you know this?
1
Not at all
2
3
4
5
Perfectly
156
Q

QU’est-ce que la MTHFR?

Quel est son role?

Décrivez les différents polymorphisme du gene de la MTHFR et l’impact sur le cycle des folates et les conséquences cliniques possibles.

A

Methylene tetrahydrofolate reducatse

Recycler la Methylene tetrahydrofolate en methyl tetrahydrofolate dans le cycle des purines des folates.

On peut etre porteurs des alleles suivants:
CC
TT
CT

How well did you know this?
1
Not at all
2
3
4
5
Perfectly
157
Q

Décrivez les différents polymorphisme du gene de la MTHFR et l’impact sur le cycle des folates et les conséquences cliniques possibles.

A

On peut etre porteurs des alleles suivants:

CC:
TT
CT

femmes porteuses de l’allele T vont avoir plus danomalies du tube neural.

Si on a les 2 T cest encore pire.
Gens plus susceptibles de manifester des signes de carence en folate.

How well did you know this?
1
Not at all
2
3
4
5
Perfectly
158
Q

Folate:

Comment peut on mitiger les effets d’un polymorphisme qui rend la MTHFR moins efficace?

A

prendre bcp d’acide folique. ou de methly THF.

How well did you know this?
1
Not at all
2
3
4
5
Perfectly
159
Q

Décrivez l’AMT pour les folates? (3 points:)
quantité
quel est l’effet deletere principal
quelles personnes sont a risque?

A

Excès
AMT= 1g /jour acide folique synthétique
Effet délétère: masquer une carence en B12
Personnes à risque: Personnes âgées

How well did you know this?
1
Not at all
2
3
4
5
Perfectly
160
Q

Quels sont les 3 risques de l’enrichissement en folate?

Quel pays a retardé l’implantation de l’enrichissement pour cette raison??

A
  1. MASQUER UNE CARENCE EN VITAMINE B12 ET ACCÉLÉRATION
    DES MANIFESTATIONS NEUROLOGIQUES
  2. AUGMENTATION CAS CANCERS COLORECTAUX
    (4-6 CAS/100 000 SOIT 15000 NOUVEAUX CAS)
    **DISTINCTION IMPORTANTE AU NIVEAU DES ÉTUDES
    OBSERVATIONNELLES (PROTECTEUR CONTRE LE CANCER)
    VS ÉTUDES D’INTERVENTION (PRO-CANCÉRIGÈNE)
  3. AUGMENTATION DU RISQUE DE GROSSESSES GEMELLAIRES
    >32% (ÉTATS-UNIS) VS >16% (GRANDE-BRETAGNE)
How well did you know this?
1
Not at all
2
3
4
5
Perfectly
161
Q

Folates et cancer
Carence Folates ___-cancérigène
Excès Folates ___-cancérigène
Folates (diète) ___-cancérigène

A

Folates et cancer
Carence Folates Pro-cancérigène
Excès Folates Pro-cancérigène
Folates (diète) Anti-cancérigène

How well did you know this?
1
Not at all
2
3
4
5
Perfectly
162
Q

Interactions médicamenteuses
Méthotrexate: antifolates

Sur quelle réaction du cycle des folates le médicament agit il a forte dose et la conséquence générale sur l’organisme?

Pourquoi ce médicament est il donc utilisé en chimiotherapie?

Pourquoi cela a donc de graves effets secondaires?

Pourquoi évite on les supplements d’acide folique quand on prend ce médicament en chimiotherapie?

A

Inhbie le recyclage de methylene THF en DHF puis en THF. Nuit donc a la synthese d’ADN.

on inhibe le cycle des folates, on inhibe la production d’ADN des cellules cancereuses

(mais ca va aussi faire des problemes aux cellules normales) et cest par exemple pourquoi c adonne des troubles GI car ces cellules sont a renouvellement rapide.

On ne donne pas de supplementaiton dacide folique en chimiotherapie pour cette raison, car ça bypasserait leffet du medicament si on en a eaucoup.

How well did you know this?
1
Not at all
2
3
4
5
Perfectly
163
Q

Interactions médicamenteuses
Méthotrexate: antifolates

Quels sont les effets a faibles doses de ce medicament? Comment gere on l’acide folique dans ces cas?

A

ce medicament est immunodepresseur, on le prend souvent pour l’arthrite rhumatoide. peut etre pris pour la maladie de crohn mais il y a des meilleures options de nos jours.

en arthrite rhumatoide, on ne veut pas bloquer la synthese d’ADN, on veut les autres effets immunosupresseurs, alors on va supplement l’ACIDE folique pour ces patients. donc dans les dossiers on va voir le supplement dacide folique souvent avec ce mediacemnt, pour reactiver le cycle.

How well did you know this?
1
Not at all
2
3
4
5
Perfectly
164
Q

b12

Faites les bonnes associations

Méthylcobalamine
Adénosylcobalamine
Hydroxycobalamine
Cyanocobalamine

avec

Forme synthétique et inactive, forme circulante
Disponible en suppléments

Forme active prédominante dans mitochondrie

Forme active prédominante dans cytoplasme
Disponible en suppléments

Forme circulante

A

Méthylcobalamine Forme active prédominante dans cytoplasme
Disponible en suppléments

Adénosylcobalamine Forme active prédominante dans mitochondrie

Hydroxycobalamine Forme circulante

Cyanocobalamine Forme synthétique et inactive, forme circulante
Disponible en suppléments

How well did you know this?
1
Not at all
2
3
4
5
Perfectly
165
Q

b12

Schéma de Digestion, Absorption, Transport, Excrétion, Utilisation (page 34)

Pourquoi a on besoin d’un estomac intact pour bien digérer et absorer la b12? (2 raisons principales)

A

car elle est prise dans les proteines alimentaires. b12 souvent dans le regne animal, on doit digerer les proteines pour la liberer. et les prots commencent a etre digéré dans lestomac grace a l’acidité et la pepsine qui la clive.
donc ces 2 fonctions libere la b12 de son lien proteique.

aussi estomac super importnat, cellules parietales secrete le facteur intrinseque qui continue son chemin,

How well did you know this?
1
Not at all
2
3
4
5
Perfectly
166
Q

b12

Schéma de Digestion, Absorption, Transport, Excrétion, Utilisation (page 34)

quel est le role de la salive?

A

la salive produit l’haptochoryne, (R binder TCI sur le shcéma)
elle resiste a l’acidite gastrique et lie la b12 digérée lors de la digestion des protéines. elle lie aussi la b12 prie en supplements.

How well did you know this?
1
Not at all
2
3
4
5
Perfectly
167
Q

Vrai ou faux, la b12 en supplement a besoin des propriétés de digestion de protéines de l,estomac pour etre digéré et absorbé

A

faux la b12 de supplement na pas besoin de cette etape.

How well did you know this?
1
Not at all
2
3
4
5
Perfectly
168
Q

b12

Schéma de Digestion, Absorption, Transport, Excrétion, Utilisation (page 34)

Que se passe t’il dans le duodenum? (2 choses principales)

A

dans le duodenum, l’haptochorine libere la b12, et la b12 se lie au facteur intrinseque. les proteases pancréatiques continuent de libérer certaines molecules de b12

How well did you know this?
1
Not at all
2
3
4
5
Perfectly
169
Q

b12

Schéma de Digestion, Absorption, Transport, Excrétion, Utilisation (page 34)

Pourquoi c’est long avant d’avoir une carence en b12 considérant le cycle enterohepatique?

A

la vitamine ou cest le plus efficace.
la b12 est libérée dans la bile et est efficacement réabsorbée, cest pourquoi ca peut etre long d’avoir une carence en b12.

How well did you know this?
1
Not at all
2
3
4
5
Perfectly
170
Q

b12

Schéma de Digestion, Absorption, Transport, Excrétion, Utilisation (page 34)

Dans quelle partie du tube digestif la b12 est elle réabsorbée?

A

iléon

How well did you know this?
1
Not at all
2
3
4
5
Perfectly
171
Q

b12

Schéma de Digestion, Absorption, Transport, Excrétion, Utilisation (page 34)

Décrivez l’Absorption de la b12 au niveau de l’enterocyte de l’iléon. nommez les protéines importantes. Décrivez le transport de la b12 et ce qui se passe avec le facteur intrinseque.

A

dans l’iléon ici, il y a un recepteur du facteur intrinseque (cubiline) qui lie le FI et ca va faire l’endocytose.

cest libéré dans l’enterocyte et ca se lie a la transcobalamine (prot) qui va passer en circulation. Le FI lui est dégradé dans l’enterocyte.

How well did you know this?
1
Not at all
2
3
4
5
Perfectly
172
Q

b12

Schéma de Digestion, Absorption, Transport, Excrétion, Utilisation (page 34)

Quel est le pourcentage d’absoprtion de la b12?

A

50% cest le porucentage d’absoprtion de la b12 alimentaire. supplements cest plus

How well did you know this?
1
Not at all
2
3
4
5
Perfectly
173
Q

b12

Schéma de Digestion, Absorption, Transport, Excrétion, Utilisation (page 34)

Combien de temps prends on pour digérer la b12?

A

4 h cest le temps pour absorber la b12.

How well did you know this?
1
Not at all
2
3
4
5
Perfectly
174
Q

b12

Schéma de Digestion, Absorption, Transport, Excrétion, Utilisation (page 34)

Une fois en circulation, que se passe til avec la b12? (indice: tissus cibles, entrée dans la cellule, fonctions de la b12)

A

va aux tissus cibles (moelle osseuse) ou cerveau, ou au foie pour stockage. reconnu par le recepteur de la transcobalamine, il y a endocytose, la b12 est libérée pour faire ses fonctions.

la MeCbl joue le role dans le cycle des folates et l’adoCbl va dans la mitochondrie pour autres fonctions.

How well did you know this?
1
Not at all
2
3
4
5
Perfectly
175
Q

Quelle est la principale fonction de la b12?

A

cofacteur de l’enzyme qui fait le methionine. (dans le cycle des folates)

How well did you know this?
1
Not at all
2
3
4
5
Perfectly
176
Q

Quelle est la 2e fonction importante de la b12?

Quelle forme de b12 est impliquée?

Pour quels tissus cette fonction est importante?

Que se passe t’il en cas de carence en b12 pour ce qui est de cette fonctioN?

Qu’est-ce que la myéline et quel est son role?

A

Co-facteur de la méthylmalonyl-coenzyme A mutase

ADENOSYL-B12

important au niveau du cerveau et de la transmission nerveuse.

Accumulation de L-MÉTHYLMALONYL-CoA qui CONDUIT À UN MÉTABOLISME ANORMAL
DES A.G CHAINE RAMIFIÉE
ET A.G. # IMPAIR DE CARBONES ce qui fait une MYÉLINATION ANORMALE

gaine de myéline autour du nerf permet la transmission nerveuse de maniere normale. important au niveau du cerveau et de la transmission nerveuse.

How well did you know this?
1
Not at all
2
3
4
5
Perfectly
177
Q

b12 et ANREF

quelles sont les valeurs recommandées?

A

Hommes-
Femmes
>14 ans
BME 2 ug

Grossesse et
Allaitement
BME 2,2 ug

Hommes-
Femmes
>14 ans
ANR 2,4 ug

Grossesse et
Allaitement
ANR 2,6 ug

How well did you know this?
1
Not at all
2
3
4
5
Perfectly
178
Q

b12 et ANREF

Quels sont les indicateurs choisis ? Pourquoi?

A

Niveau normal de cobalamine sérique
Profil hématologique normal

veut dire qu’il n,y a pas d’anomalie megaloblastique.

et maintenir la cobalamine serique.

How well did you know this?
1
Not at all
2
3
4
5
Perfectly
179
Q

b12

Pour quel sous groupe de la population recommande on que les ANR soit comblés par des formes synthetiques? Pourquoi?

A

> 50 ans: atrophie gastrique survient chez 10-30% individus:

même BME/ANR mais idéalement provenant de forme synthétique

How well did you know this?
1
Not at all
2
3
4
5
Perfectly
180
Q

Quel est l’AMT pour la b12?

A

Aucune toxicité associée: pas d’AMT

How well did you know this?
1
Not at all
2
3
4
5
Perfectly
181
Q

Vrai ou faux, la déficience en b12 apparait tres rapidement.

A

Faux Prend beaucoup de temps à apparaître;
Premiers signes (biochimiques) apparaissent des
mois voire des années après l’apparition de la cause
de la déficience.

How well did you know this?
1
Not at all
2
3
4
5
Perfectly
182
Q

Comment nomme ton l’anémie liée a une perte du facteur intrinseque?

Combien de temps peut elle prendre a se developper?

Pourquoi est-ce long a se developper?

A

Perte complète de FI: peut prendre 3-5 ans
avant d’être carencé.
Prend beaucoup de temps à apparaître;
Premiers signes (biochimiques) apparaissent des
mois voire des années après l’apparition de la cause
de la déficience.

How well did you know this?
1
Not at all
2
3
4
5
Perfectly
183
Q

déficience en b12

Quels sont les signes biochimiques?

A

Direct: Cobalamine sérique (diminue)

Indirects: Homocystéine sérique (augmente)

Acide méthylmalonique urinaire (vient du methylmalonyl coa, ce substrat saccumule en cas de carence, ca vetre excreter par le rein, sous la forme acide methlymalonique urinaire. )

How well did you know this?
1
Not at all
2
3
4
5
Perfectly
184
Q

déficience en b12

Quels sont les signes hematologiques?

A

Anémie mégaloblastique:
Manifestations identiques
à celles de la carence
en folates

How well did you know this?
1
Not at all
2
3
4
5
Perfectly
185
Q

déficience en b12

Quels sont les signes neuro-psychiatriques?

A

Peuvent apparaître après l’anémie:

  1. Myélopathie: Perte de myéline
  2. Neuropathie: perte proprioception, vibration
  3. Troubles cognitifs (psychose, perte de mémoire)
  4. Ataxie, incontinence
How well did you know this?
1
Not at all
2
3
4
5
Perfectly
186
Q

déficience en b12

Quels sont les 3 catérogies d’indicateurs que l’on peut regarder?

A

Signes
biochimiques

Manifestations
hématologiques

Manifestations
Neuro-psychiatriques

How well did you know this?
1
Not at all
2
3
4
5
Perfectly
187
Q

déficience en b12

Quels sont les 3 grandes causes de déficience?

A

Apports Absorption Métabolisme

How well did you know this?
1
Not at all
2
3
4
5
Perfectly
188
Q

déficience en b12

QUelle est en generale la cause d’un apport faible?

A

Végétalisme
Enfants de mères
végétalistes

How well did you know this?
1
Not at all
2
3
4
5
Perfectly
189
Q

déficience en b12

Décrivez les problemes d’absopriton qui peuvent se produire.

(indice: peut etre different pour la b12 libre ou alimentaire)

A
Pour la b12 libre et la b12 alimentaire
Anémie pernicieuse
Gastrectomie (30% cas)
Maladies iléales
Pullulation bactérienne
Seulement pour la b12 alimentaire
Gastrectomie (50% cas)
Atrophie gastrique
Chirurgies gastriques
Inhibiteurs acidité gastrique
How well did you know this?
1
Not at all
2
3
4
5
Perfectly
190
Q

déficience en b12

Décrivez les problemes de transport qui peuvent se produire.

A

Absence de
Transcobalamine II
(maladie génétique)

How well did you know this?
1
Not at all
2
3
4
5
Perfectly
191
Q

Existe-t-il des sources non-animales de vitamine B12 ?

A

on peut manger la levure alimentaire, dans certaines algues (la nori), et la chlorela. les boissons végétales parfois sont enrichi en b12. (surtout soya).

les bactéries (cyanobactéries) en font aussi, ils produisent une b12, (ex: spiruline) elle n’est pas active. cest un succédané de b12.

les aliments fermentés comme le tempeh et nato ont de la b12. parce quelles sont fermentées.

How well did you know this?
1
Not at all
2
3
4
5
Perfectly
192
Q

b12 : Déficience infra clinique (insuffisance)

Quels sont les marqueurs principaux?

Pourquoi doit on faire attention avec les personnes agées?

A

Portrait clinique:

  • Cobalamine sérique abaissée
  • Homocystéine élevée
  • *Répond à l’administration de cobalamine
    • Absence de signes cliniques
    • 10-15 % Personnes âgées
How well did you know this?
1
Not at all
2
3
4
5
Perfectly
193
Q

Choline:

Faites les bonnes associations

Choline

Phosphatidylcholine
Sphingomyéline

Acétylcholine

Betaine

avec

Neuromédiateur

Composantes des membranes cellulaires
et des lipoprotéines (rôle dans transport
des lipides)

Métabolite de la choline, sert de donneur de méthyl

Forme libre, circulante

A

Choline Forme libre, circulante

Phosphatidylcholine
Sphingomyéline Composantes des membranes cellulaires
et des lipoprotéines (rôle dans transport
des lipides)

Acétylcholine Neuromédiateur

Betaine Métabolite de la choline, sert de donneur de méthyl

How well did you know this?
1
Not at all
2
3
4
5
Perfectly
194
Q

Quels sont les principaux roles de l’acetyl choline? (4)

A

Mémorisation

Coeur:
Bradychardie

Vaisseaux:
vasodilatation

Glandes salivaires,
Intestin, estomac:
SÉCRÉTION

195
Q

Quels sont les 2 precurseurs de l’acetyl choline?

A

Phosphatidylcholine et acetyl CoA

196
Q

Pour quelle maladie notoire le role de l’acetylcholine a til été étudié? Est-ce que les resultats ont été concluants?

A

dans quelle maladie la choline a été tres étudiée?
Alzheimer.

joue un role dans la memorisation. est-ce que ca protege contre l’alzheimer?? c’Est pas concluant.

197
Q

Quel est le role de la betaine?

Dans quel cas est-elle tres sollicitée?

Comment ces besoins vont donc varier?

A

betaine es tun donneur de methyl comme la THF

cest une voie secondaire de transformation de l’homocysteine en methionine. voie tres solicitée lorsque lon manque de folate et de b12. Les besoins en choline vont etre tres dependnant des 2 autres vit. car on doit compenser ça.

198
Q

Que sont les ANREF en choline?

quel indicateur a été choisi?

A

Apport suffisant

AS >19 ans
Femmes 425 mg
Hommes 550 mg
Grossesse 450 mg (tous âges)
Allaitement 550 mg (tous âges)

Maintien des enzymes hépatiques normales

199
Q

Quels sont les enzymes hépatiques normalement
mesurés dans le sang et qui permettent d’évaluer la
fonction hépatique?

Dans quel cas ces enzymes se retrouvent plus en circulation?

A

AST et ALT, l’alanine transferase et l’aspartate transferase (AST)

aussi la gamma ggt (glutamyl transferase)

ces enzymes augmentent quand il y a lesion d’Hepatocytes.( integrité des hepatocytes) ca libere ces enzymes dans le sang. en clinique cest un signe de dommage hepatique.

200
Q

Vrai ou faux, les carences en choline peuvent causer des lesions au foie

A

carence en choline mene a des lesions hepatiques,

201
Q

Quels sont les symptomes de deficiences en choline?

A

Stéatose
Anomalies enzymes hépatiques
Hépatocarcinome

202
Q

Pourquoi la deficience en choline cause la stéatose?

A

si carence en phosphatidyl choline dans le foie, le VLDL n’est pas bien fait, ca affecte son fonctionnement.

203
Q

Quel est l’AMT pour la choline? Comment l’a ton fixer?

A

Excès:
AMT= 3,5g /jour
Effet délétère: Hypotension légère
Autre signe distinctif: odeur de poisson chez 1%
population américaine, causée par la triméthylamine
(métabolite de la choline)

204
Q

Les personnes véganes sont à risque de déficience en choline

A

faux choline est-ce surtout animal?
le jaune d’oeufs est tres riche en choline.
son petit nom: lecityne.
il y a de la choline dans le soya et le blé, donc pas exclusivement animal.

205
Q

Pourquoi la cellule renferme-t-elle autant de potassium?

A

La cellule renferme beaucoup d’anions tels des protéines, des phosphates, du chlore et du bicarbonate qui sont des molécules porteuses d’une charge négative. Comme le potassium est porteur d’une charge positive, il permet de maintenir l’équilibre ionique (électroneutralité) à l’intérieur des cellules.

206
Q

Comment les électrolytes traverse les membranes cellulaires?

A

Via transporteur

207
Q

Quel est le principal transporteur du sodium et potassium?

A

Pompe Na+K+ATPase

208
Q

QUel est le mecanisme de la Pompe Na+K+ATPase

A

Fait sortir du sodium et rentrer du potassium dans la cellule.

209
Q

Quelles hormones stimulent la Pompe Na+K+ATPase

A

L’insuline et l’adrénaline

210
Q

Décrivez la fonction d’absoprtion et de transport du sodium

A

Au niveau de l’intestin mais aussi au niveau du rein,

ex glucose dans l’enterocyte est couplé au sodium
meme chose au niveau de la réabsoprtion reinale. le glucose utilise des transporteurs qui sont couplés au sodium.

211
Q

Décrivez la fonction de neurotransmission du sodium.

A

Lorsque stimulus arrive, les canaux qui sont voltages dépendants vont souvrir, font entrer le sodium dans la cellule. Le potentiel de repos est tres negatif. Lentrée du sodium va rendre plus positif l’espace intracellulaire. le potentiel intra augmente, cest la depolarisation.

la cellule ouvre ensuite des canaux potassium pour repolairser la cellule.

finalement les canaux sodiques se referme pour faire une legere hyperpolarisation puis retourner au potentiel de de repos.

212
Q

Décrivez la fonction de mainiten du volume et de la pression sanguine du sodium. 4 points

Autres molecules importantes

osmolites

augmentation natremie

diminution natremie

A

Le sodium nest pas seul, par exemple pes proteines aussi et les autres osmolites.

Les osmolites modulent les echanges d’eau de part et d’autre de la membrane cellulaire.

Lorsque la natrémie augmente, l’eau intracellulaire se déplace vers l’eau extracellulaire pour rééquiliber la natremie. ca cause un retrecissement cellulaire. AUgmentation de la pression sanguine.

L’inverse se produit si il y a une baisse de la natremie. L’Eau va vers les cellules. gonflement des cellules. Pour tenter de reequilibrer la natremie. Diminution de la pression sanguine.

213
Q

Décrivez la fonction de mainiten du volume et de la pression sanguine du sodium. Quel est l’impact du systeme renine angiontensine au niveau du sodium. 3 points:

COmment le sodium enclenche ce systeme?

Quelle est la cascade de réaction qui s’en suit?

Sur quel mecanisme l’angiotensine 2 affecte la natrémie?

A

ce systeme peut etre affecté par une baisse de la natremie.

Ca fait sécréter la rénine. qui va enclencher la formation des precurseurs de l’angiotensine 2 jusqua y arriver.

cette molecule favorise une reabsoprtion deau et de sodium. On favorise aussi la sortie de potassium.
Fait produire l’aldosterone. qui stimule L,excretion de potassium et la réabsorption d’eau. Agit aussi sur l’hypophyse pour faire l’ADH. qui agit aussi sur les tubules qui font réabsorber l’eau. Tout dans le but de faire la réabsortipion d’eau et augmenter la pression et le volume sanguin.

214
Q

sodium ANREF (2019)

QUels sont les 3 indicateurs pour fixer les ANREF?

A

Éviter les carences en général car les bas apports de sodium sont associés a une pauvre diete.

Remplacer les pertes sodiques
(lors de situations extrêmes: To élevée, sudation chez personnes actives)

215
Q

sodium ANREF (2019)

Pourquoi a ton aboli l’AMT? par quoi l’a on remplacé?

A

AMT en general on parle de toxicité, mais on a pas vrm d’effets toxiques, cest plutot une association avec les maladies chroniques. C’est donc un apport lié a une redicutiion du risque de maladie chronique.

216
Q

sodium ANREF (2019)

Quel est l’AS?

A

1.5 g par jour

217
Q

sodium ANREF (2019)

Quels groupes ont des recommandations speciales?

A

Travailleurs exposés à une chaleur extrême
Athlètes endurance

Pourrait avoir des besoins superieurs mais cest du cas par cas.

218
Q

Les apports en sodium de la diète nord-américaine proviennent majoritairement du sel ajouté lors de la cuisson et de la consommation des aliments (sel ajouté à table).
Vrai Faux

A

Faux Les apports en sodium de la diète nord-américaine proviennent à plus de 75% des aliments transformés. Environ 5% de nos apports proviennent du sel ajouté à la cuisson et 6% du sel ajouté lors de la consommation des aliments. Le reste du sodium consommé dans la diète nord-américaine provient de sources naturelles de sodium soit des aliments qui ont une teneur élevée en sodium. Le lait, les viandes, les fruits de mer et certains légumes contiennent naturellement du sodium.

219
Q

Quels sont les causes de l’hyponatrémie? (3)

A

DIminution des Apports (très rare)
Augmentation des Pertes (diarrhées, vomissements, diurétiques)
Ingestion excessive d’eau (effet de dilution)

220
Q

Quels sont les conséquences de l’hyponatrémie? 1 effet principal et 3 symptomes .

A
Surhydratation
cellulaire:
Oedème cellulaire
Convulsions
Coma
221
Q

Quels sont les causes de l’hypernatrémie? (3)

A

Augmentation de Concentration osmotique du sang
Augmentaion des Apports
Augmentation des Pertes d’eau, déshydratation globale

222
Q

Quels sont les conséquences de l’hypernatrémie? (1 effet principal et 4 symtpomes)

A
Déshydratation
cellulaire:
Soif
Fatigue
Crampes musculaires
Spasmes
223
Q

Potassium:

Quelles sont les 2 sources dans l’alimentation? Sous quelle formes se trouve il selon la source?

A

Dans les aliments: K est complexé:

  • Citrate
  • Phosphate

Dans les suppléments:
-Chlorure de K

224
Q

Pourquoi le potassium alimentaire est associé a des effets basifiants?

Quel est l’avantage physiologique quand on a une diete riche en proteines par exemple?

A

Du a son complexe avec le citrate.
qui régénère le bicarbonate. ça va augmenter le pH.

ca permet de compenser pour les dietes riches en proteines (AA soufrés) les viandes ou les céréales qui font une charge acide nette. Ca permet de proteger l’os car sinon il devrait se résorber pour fournir le tampon.

225
Q

Potassium:

Pourquoi les études montrent qune alimentation riche en potassium permet davoir des os en santé?

A

Parce qune alimentation riche en potassium alimentaire fait rentrer bcp de citrates, ça fournit des tampons, et previent l’os de devoir se résorber pour fournir ces memes tampons

226
Q

Potassium: ANREF (2019)

Quel est l’indicateur choisi?

Comment etait l’AS avant 2019

QUel est l’AMT pour le potassium?

A

Apports médians de la population nord-américaine

avant plus élevé, car différent indicateur. les conaissances étaient différentes.

Pas d’AMT

227
Q

Quel groupe de la population doit faire attention avec les supplements de potassium?

A

Mais… précaution avec les suppléments de K+ car risque d’arythmie chez ceux à risque de
pathologies cardiaques, rénales et ceux qui prennent des médicaments causant une
rétention rénale de K.

228
Q

Pourquoi les gens ayant des pathologies reinales doivent faire attention aux supplem,ents de K+?

A

Le rein est responsable de lhomeostasie du potassium, ca peut affecter lequilibre si on donne un supplement.

229
Q

Quels sont les 2 types d’hypokaliemies?

A

Déficience
franche

Insuffisance

230
Q

Quels sont les symptomes de déficience franche en potassium?

A

HYPOKALIÉMIE NORMOKALIÉMIE
Faiblesse musculaire
Arythmie cardiaque
Intolérance au glucose

231
Q

Quels sont les symptomes d’insuffisance en potassium?

A
NORMOKALIÉMIE
Augmente Pression sanguine
Augmente Sensibilité au sel
Augmente Risques de néphrolithiases
Augmente Remodelage osseux
232
Q

Pourquoi la carence en potassium peut causer une intolérance au glucose?

A

Insuline est mal sécrétée

233
Q

Pourquoi l’insuffisance en potassium peut causer le remodelage osseux et augmenter le risque d’ostéoporose

A

Pas assez de potassium alimentaire pour l’effet tampon.

234
Q

Pourquoi l’insuffisance en potassium peut causer une augmentation de la pression sangunet et la sensibilité au sel?

A

Le corps veut réabsorber le potassium, mais en faisant cela on augmente la réabsoprtion de sodium, ca active les canaux sodiques dans le rein qui réabsorbe le sodium. Donc cette augmentation de sodium augmente la P sanguine et fait que la P devient tres sensible au sodium (va augmenter bcp quand on consomme du sodium)

235
Q

Pourquoi l’insuffisance en potassium peut causer les pierres aux reins?

A

Aliments plus acidifiants favorisent la resopriton osseuse, augmente le calcium dans le sang et augmente les pertes urinaires de calcium.

Le citrate de K aumgente les pertes urinaires de citrates. LE citrate chelate le calcium et l’empeche de precipiter.

donc lalimentation riche en potassium protege donc des pierres aux reins.

donc en insuffisance, le calcium se dépose plus et le pH est plus acide donc il y en a plus du a la résorption osseuse.

236
Q

Eau:

Comment se distribue l’eau dans le corps? (2 grans compartiments, dont 1 qui a 3 sous compartiments)

A

Compartiment
extracellulaire: 40%

se divise en 3

Liquide
Interstitiel
28% (autour des capillaires, autour de l’espace extracellulaire).
Plasma
8%
Transcellulaire
4% (salive, larmes, eau autour des cellules).

Compartiment
intracellulaire: 60%

237
Q

Comment l’eau se deplace du compartiemnet intra vers extra cellulaire

A

Via les canaux nommés aquaporines, ils controlent les echanges d’eau.

238
Q

eaue

Quel compartiment est impacté par l’oedeme?

A

Linquide interstitiel

239
Q

Quels sont les facteurs principaux qui impactent le contenu en eau du corps?

A

Age : plus on est jeune plus on a d’eau

Sexe: hommes ont plus d’eau

240
Q

Pourquoi le volume corporel d’eau est-il plus faible chez la femme?

En raison de la sécrétion d’estrogènes plus importante chez la femme
En raison de la composition corporelle qui diffère entre les hommes et les femmes
En raison du volume urinaire qui est plus élevé chez les femmes
En raison des pertes menstruelles

A

En raison de la composition corporelle qui diffère entre les hommes et les femmes
Bonne réponse.
Les femmes ont un pourcentage de graisse plus élevé que les hommes qui eux, ont un pourcentage de masse musculaire plus élevé. La graisse ne renferme pas d’eau. Les muscles en renferment une quantité non-négligeable. C’est cette différence de composition corporelle qui explique pourquoi les femmes ont un plus volume corporel d’eau plus faibel que celui des hommes.

241
Q

Nommer les facteurs de pertes hydriques et ce qui les faits varier. (4)

A

Excrétion urinaire
Elle varie selon la consommation d’eau et selon la quantité
de soluté à excréter

Évaporation cutanée
Elle varie beaucoup selon le climat

Excrétion par les poumons
Elle varie selon le climat et l’altitude

Excrétion fécale

242
Q

Nommez les facteurs de production d’eau endogene

A

Production métabolique
L’oxydation de 1 g de glucide - 0,6 g d’eau
L’oxydation de 1 g de protéine - 0,42 g d’eau
L’oxydation de 1 g de lipide - 1 g d’eau

243
Q

Bilan hydrique : faites les bonnes associatons

urinaire
cutanée
par les poumons
fécale
métabolique

avec

pertes 100-200
Production de 250-350
pertes 500-1000
pertes 250-350
pertes 450-1 900
A
urinaire pertes 500-1000
cutanée  pertes 450-1 900
par les poumons perte 250-350
fécale pertes pertes 100-200 
métabolique Production de 250-350
244
Q

Bilan hydrique : faites les bonnes associatons

L’oxydation de 1 g de glucide -
L’oxydation de 1 g de protéine -
L’oxydation de 1 g de lipide -

avec

0,42 g d’eau
0,6 g d’eau
1 g d’eau

A

L’oxydation de 1 g de glucide - 0,6 g d’eau
L’oxydation de 1 g de protéine - 0,42 g d’eau
L’oxydation de 1 g de lipide - 1 g d’eau

245
Q

Eau et Anref

Comment a ton établi les ANREF?

A

Indicateur: Maintenir l’hydratation normale
AS: apport médian

246
Q

Eau et Anref

Quel est l’AMT?

A

Pas d’AMT mais la capacité rénale maximale d’excrétion 0.7L/h

247
Q

Eau et Anref

QUels sont les AS? (en l par jour)

Pour les hommes, femmes, grossesse, et allaitement

A

Hommes 14-18 et plus que 19 ans

3.7

Femmes 14-18 et plus que 19

2.7

Grossesse

3

Allaitement

3.8

248
Q

Charte de couleurs urinaires pour athlètes

Quelle couleur indique qu’il est bien hydraté? Déshydraté

est-ce que ça s’applique aux non ahtletes?

A
Pale = hydraté
foncé = déshydraté

oui

249
Q

Qu’est-ce que l’urobiline?

A

Saviez-vous que?

L’urine contient de l’urobiline, un pigment qui lui donne sa couleur jaune. Une diminution de l’excrétion urinaire d’eau entraîne la concentration de ce pigment et donc une urine plus foncée. Bien entendu, l’urobiline n’est pas le seul pigment excrété dans l’urine, certains pigments contenus dans les aliments peuvent aussi modifier la couleur de l’urine (ex: betteraves).

250
Q

Nommez les 5 principaux facteurs de variation des pertes hydriques?

A
  • 1- CHALEUR ET ACTIVITÉ PHYSIQUE:
  • 2- ALTITUDE ET FROID:
  • 3- MACRONUTRIMENTS:
  • 4- CAFÉINE:
  • 5- ALCOOL:
251
Q

En quoi les macronutriments peuvent influencer les pertes hydriques (3 facteurs principaux)

A
  • 3- MACRONUTRIMENTS:
  • PROTÉINES (augmente pertes pour excrétion urée)
  • DÉFICIT DE GLUCIDES (Augmente pertes pour excrétion corps cétoniques)
  • FIBRES (Augmente pertes fécales d’eau)
252
Q

En quoi la chaleur et l’Activité physique peuvent influencer les pertes hydriques

A
  • 1- CHALEUR ET ACTIVITÉ PHYSIQUE: Augmente Sudation

* * BESOINS DOUBLENT À 30 C ET TRIPLENT À 40 C

253
Q

En quoi l’ALTITUDE ET FROID peuvent influencer les pertes hydriques

A

-2- ALTITUDE ET FROID: (Augmente perte eau pulmonaire, Augmente diurèse)

254
Q

En quoi la caféine peuvent influencer les pertes hydriques

A

-4- CAFÉINE: Effets variables à doses physiologiques

255
Q

En quoi l’alcool peuvent influencer les pertes hydriques

A

-5- ALCOOL: Effet diurétique transitoire

256
Q

Expliquer pour la soif s’installe quand on mange salé. (long développement)

A

AUgente la natrémie et l’osmolalité.

Le corps n’Aime pas ça. ça le désiquilibre.

Il va faire sortir de l’eau du milieu intra vers extra.

CAuse une deshydratation cellulaire au profit du milieu extracellulaire.

Tout ça va activer des recepteurs sensibles au changement d’osmolarité. ou les barorecepteurs sensibles au changement de volume.

ces recepteurs sont a l’hypothalamus.

Ils sont activés par ces changements qui envoie un message a l’hypophyse pour qu’elle secrete l’ADH, qui agit sur le rein pour permettre la retention d’eau et L’effet de soif lui s’active lorsque les recepteurs hypothalamiques sont activés.

257
Q

Quels sont les effets de la carence en eau en fonction du pourcentage du crops humain perdu en fluide.

A

2%, on a soif

4%, diminue la force musculaire et l’endurance (on le voit dans les epreuves d,endurance.

10% faiblesse musuclaire, tolerance a la chaleur diminue, confusion mentale.

20% de perte, Coma et Mort.

258
Q

Excès d’eau

qui sont les sous groupes a risques?

Quelles sont les consequences??

A
  • NOURRISSONS
  • TROUBLES PSYCHIATRIQUES (POLYDIPSIE PSYCHOGÈNE)
  • USAGE DE MÉDICAMENTS PSYCHOTROPES
  • ÉPREUVES D’ENDURANCE SOUTENUE

**>20 L PAR JOUR: LÉSIONS IRRÉVERSIBLES DE LA VESSIE:
AMINCISSEMENT MUSCLES VESSIE
RETARD DES PERCEPTIONS SENSORIELLES AU NIVEAU VESSIE
ALTÉRATION DÉBIT URINAIRE

259
Q

Quelle est l’une des particularités du cuivre?

Il s’agit du minéral dont les besoins pour le corps sont les plus élevés

Il s’agit du seul minéral à ne pas avoir besoin de transporteurs pour son absorption intestinale

Il s’agit du seul minéral qui circule librement dans le sang

Il s’agit de l’un des rares minéraux à être excrété dans la bile

A

Il s’agit de l’un des rares minéraux à être excrété dans la bile

260
Q

Lequel des énoncés suivants est faux?

La céruloplasmine peut oxyder l’ion ferreux (Fe2+) en ion ferrique (Fe3+)

De fortes doses de fer diminuent l’absorption du cuivre.

Un cuproenzyme est impliqué dans la synthèse de mélanine

La céruloplasmine est une protéine de transport du cuivre dans le sang

A

De fortes doses de fer diminuent l’absorption du cuivre.

261
Q

Tous les signes suivants sont associés à la carence en cuivre à l’exception de:

Anémie

Démyélination

Dépigmentation

Insuffisance rénale

A

Insuffisance rénale

262
Q

Quel critère a été retenu pour établir le BME/ANR en sélénium?

L’activité optimale de la iodothyronine deiodinase

Les apports médians en sélénium de la population américaine

L’activité optimale de la thioredoxine reductase

L’activité optimale de la glutathion peroxydase

A

L’activité optimale de la glutathion peroxydase

263
Q

Quel est l’énoncé qui définit le mieux la maladie de Keshan?

Cardiomyopathie causée par une infection virale et exacerbée par une carence en sélénium

Maladie neurologique causée par une carence en sélénium

Maladie cardiaque causée par une carence en sélénium qui est particulièrement prévalente en Amérique du Nord

Cardiomyopathie résultant d’une carence en sélénium combinée à une carence en zinc

A

Cardiomyopathie causée par une infection virale et exacerbée par une carence en sélénium

264
Q

La carence en chrome peut mener à un/une:

Altération du système reproducteur

Intolérance au glucose

Insuffisance pancréatique

Maladie neurologique

A

Intolérance au glucose

265
Q

Lequel des énoncés est exact?

La xanthine oxydase est une enzyme qui contient du molybdène

Le chrome VI est la forme retrouvée dans les aliments

L’apport médian de la population américaine a servi d’indicateur pour établir l’apport suffisant en molybdène

La toxicité en chrome est associée à une altération du système reproducteur chez le rat

A

La xanthine oxydase est une enzyme qui contient du molybdène

266
Q

Parmi les individus suivants, lesquels ont des besoins particulièrement élevés en iode?

Les femmes enceintes

Les enfants en croissance

Les femmes adultes

Les garçons adolescents

A

Les femmes enceintes

267
Q

Lequel des énoncés est vrai?

La sécrétion hypophysaire de TRH est stimulée en réponse à une baisse de la concentration circulante d’hormones thyroïdiennes

La sécrétion hypophysaire de TSH est stimulée en réponse à une baisse de la concentration circulante d’hormones thyroïdiennes.

L’iode n’a pas d’AMT

Une carence en iode qui se développe chez l’enfant en croissance conduit au crétinisme

A

La sécrétion hypophysaire de TSH est stimulée en réponse à une baisse de la concentration circulante d’hormones thyroïdiennes.

268
Q

Le crétinisme se caractérise par:

De la surdité

Des crises d’épilepsie réfractaires au traitement

De l’hypogonadisme

Une hyperthyroïdie

A

De la surdité

269
Q

Parmi les signes cliniques suivants, lequel ou lesquels pourrai(en)t être associé(s) à une carence en manganèse?

Hypercholestérolémie

Intolérance au glucose

Retard de croissance

Anomalies squelettiques

A

Parmi les signes cliniques suivants, lequel ou lesquels pourrai(en)t être associé(s) à une carence en manganèse?

Retard de croissance
Bonne réponse.
Ce signe clinique de carence s’explique par l’implication du manganèse dans la synthèse du sulfate de chondroïtine, une composante du cartilage nécessaire à sa croissance et son développement.

Anomalies squelettiques
Bonne réponse.
Comme le manganèse est impliqué dans la synthèse de sulfate de chondroïtine, une composante du cartilage, une carence en cet oligo-élément peut entraîner des anomalies squelettiques.

Intolérance au glucose
Bonne réponse.
Ce signe de carence s’explique par le rôle joué par le manganèse dans le métabolisme des glucides.

270
Q

Lequel des énoncés suivants est vrai?

La rétention du fluor par le corps est particulièrement efficace chez les enfants

La prévention de la fluorose est l’indicateur qui a permis de fixer l’apport suffisant en fluor

Seuls les os et les dents retiennent du fluor dans l’organisme

Le fluor fragilise l’émail des dents à l’âge adulte

A

La rétention du fluor par le corps est particulièrement efficace chez les enfants

271
Q

Lequel des éléments suivants n’est pas considéré comme un ultra-trace?

Arsenic

Cuivre

Nickel

Bore

A

Cuivre

272
Q

lequel des ultra-trace suivants exerce une action insulinomimétique?

Bore

Arsenic

Silicium

Vanadium

A

Vanadium

273
Q

À voix haute décrivez le schéma d’absoprtion, transport, distribution et excrétion du cuivre.

A

Les sources sont bien reparties entre animales et vetgetales, les crustacés sont la principale, ensuite les noix et graines etc.

l’absorption est meilleur pour les sources animales, et elle diminue selon la grandeur de la dose.

Le Cu2+ (cuprique) doit etre reduit en cupreux (Cu1+) par une reductase membranaire de la membrane bordure en brosse.

Le canal CTR1 laisse entrer le cupreux dans l’enterocyte. Une fois dans l’enterocyte, il se lie a la metalothioneine, qui sert de reserve pour palier aux besoins en cuivre de l’enterocyte. A haute dose il y aura aussi diffusion de cuivre vers l’enterocyte.

La protéine ATP7A véhicule le cuivre vers la membrane basolaterale et devient un canal pour laisser passer le cuivre en circulation.

Une fois dans la circulation, il se lie a la transcupreine. Il peut aussi se lier a l’albumine. ça va vers le foie.

Le foie est important, il entrepose le cuivre et produit la ceruloplasmine et l’albumine qui vont amener le cuivre vers les tissus cibles. Le foie produit aussi la bile qui va excreter le cuivre en trop.

Le cuivre est important dans la synthese de mediateur, notamment les os et le muscle et la synthese de collagene.

Dans l’enterocyte, l’hephaestine est importante, c’est une ferooxydase qui oxyde le fer ferreux en ferrique pour permettre son transport via la transferrine. l’hephaestine est cuivre dependante. et cest pourquoi la carence en cuivre peut causer l’anemie ferriprive.

274
Q

Quel est le role chimique principal des cuproenzymes?

A

Oxyder

275
Q

A voix haute, décrivez la maladie de Henke

A

Une mutation de la protéine ATP7A fait que le cuivre ne se rend pas de l’enterocyte a la circulation.

Toutes les cuproenzymes en sont impactées

les symptomes seront donc par exemple l’hypopigmentation et l’hypotonie.

276
Q

Nommez les principales fonctions, les cuproenzymes et les symptomes de carences associés. (7)

A
Amines oxydase (lysil oxydase)
Structure
Collagène
et élastine
=
Troubles
osseux
Amines oxydase (monoamine oxydase)
Dégradation
catécholamines
=
Troubles
Neurotransmission

Ferroxydases (ferooxydase 2 et ceruloplasmine)
Oxyde Fe2+ en Fe3+ = Anémie

Cytochrome C
oxydase
Chaîne transport
Électrons
Synthèse PL
=Démyélination 
Dopamine
Hydroxylase
Catalyse conversion dopamine
en noradrénaline = Troubles
Neurotransmission
Catalyse conversion
tyrosine en dopamine
Conduire à la synthèse de
mélanine
Catalyse conversion
tyrosine en dopamine
Conduire à la synthèse de
mélanine = Achromotrichie
(Dépigmentation)

Superoxyde
dismutase
Enzyme
Antioxydante

277
Q

Quels sont les indicateurs choisis pour les anref en cuivres (3) ?

Comment les ANREF évolue selon lage et la grossesse ou l’allaittement?

Comment l’AMT a été fixé?

A

Taux optimaux de :

1) Concentration plasmatique et plaquettaire de cuivre
2) Activité SOD érythrocytaire
3) Taux de céruloplasmine sérique

besoins varient pas en fonction du sexe, mais de l’age. variation minime. ++ pour grossesse et allaitement

lésions hépatiques

278
Q

Décrivez a voix haute la maladie de wilson

QUel est le symptome physique typique de cette maladie?

A

Mutation de la ATP7B, qui est necessaire a l’efflux de cuivre du foie et du cerveau.

Ça s’accumule et ça fait une cirrhose et de la deterioration nerveuse.

Anneau de kayser-fleischer (cuivre s’accumule autour de l’iris)

279
Q

A voix hautes, décrivez les différentes formes de selenium, et les sources.

A

Inorganique vs sélénoprotéines

Inorgaique = selenite et selenate et vient de supplkeemnts ou aliments fortitifés

Ssélénoprotéines = Sélénocystéine = animales

ou
Ssélénoprotéines = Sélénométhionine = sources vegetales

280
Q

Quelle est la fraction de methionine et cysteine qui vont integrer le selenium?

A

petite fraction

281
Q

Décrivez a voix hautes les pays ou l’on trouve plus ou moins de selenium dans l’alimentation et pourquoi

A

Amerique du nord, sols riches, on en a plus et dans les aliments et dans la viande d’elevage.

Au bresil, beaucoup dans les noix de bresil.

En chine et en asie en general il y en a peu dans le sol et on a donc des carences.

En europe il y en a peu aussi mais on en trouve certaines quantités dans les viandes et fruits de mer.

282
Q

A voix haute, résumez le schéma de la physiologie du selenium.

A

Le Selenate est absorbé a 100% par l’enterocyte

LE selenite a 50% car se complexe avec les phytates

Les selenoproteines se comportent comme des AA et vont donc etre absorbés environ a 90%.

Le selenium va au foie en reserve, les AA font dans le pool d’AA et se comporte comme tel mais peuvent etre dégradé et stocké au foie si il en manque.

Le foie libere le selenide qui est le metabolite actif du selenium.

Le selenide va faire plusieurs fonctions, dont les suivantes;

Aider aux glutathion peroxydases, (role antioxydant)
Selenoproteines p et W (role inconnu)
Iodothyronine déiodinaese (syntheses hormones thyroidiennes)
Thioredoxine reductases (regenere acide ascoribe lorsque oxydée)

283
Q

Anref selenium

Quel est l’indicateur choisi?

de quels ordres sont les besoins et comment ça évolue selon l’age et l’état?

Comment l’AMT a été déterminé?

A
Activité
optimale
de
l’enzyme
glutathion
peroxydase

pas mal pareil pour tout le monde sauf ANR augmente pour grossesse et plus pour allaitement.

presence de selenose, un signe de toxicité, donne une coloration noire a certains tissus dont les ongles.

284
Q

Selenium

Décrivez a voix haute l’Expérience du virus coxsackie

A

Expérience chez la souris

pour des souris qui ont un statut normal ou deficient en selenium

on injecte aux deux un virus non virulent ou virulent.

on se rend compte qu’un bon statut en selenium influence la virulence du virus. (+ de selenium veut dire moins de cardiopathie)

En faisant des enquetes populationelles on a decouvert que les endroits affectés par le virus sont ou il y a peu de selenium dans le sol, et quand on donne un supplement ça aide.

285
Q

Quel est le role general du molybdene?

Quelles sont les 2 plus notables fonctions?

A

Cofacteur de certaines oxydases entre autre.

Les plus notables sont la xantine oxydase cytoplasmique qui fait l’acide urique avec la xantine.

Il y a aussi la sulfite oxydase mitochondriale sui fait des sulfates avec des sulfites (donc contribue au metabolisme des AA soufrés.

286
Q

Quel est l’indicateur pour les ANREF du molybdene?

Comment les ANREF évolu selon l’Age et l’état? Quel est l’ordre de grandeur?

QUel est le critere retenu pour l’AMT?

A

Bilan a l’equilibre (perte = apports)

= entrehommes et femmes, augmente un peu avec l’age, grossesse et allaitement ++

AMT fixé selon l’altération du systeme reproducteur chez le rat.

287
Q

Quels sont les 2 formes de chromes, d’ou proviennent elles, discuter de la toxicité du chrome. et des échanges entre les 2 formes.

A

Chrome 6 vient d’usage industriel, il est facilement reduit en chrome 3, qui liui vient des grains entiers.
Le chrome n’est pas toxique.

288
Q

A voix haute, décrivez le schéma de potentialisation de l’insuline par le chrome.

A

Si le chrome est présente, il se lie a sa proteine, APOLC, qui va alors se lier au recepteur a l’insuline, et lorsque lié, ca augmente l’entrée du glucose.

289
Q

Chrome, ANREF

quel est l’indicateur pour le chrome, comment les besoins varient ils avec l’age et selon le sexe. discuter de l’AMT

A

C’est un AS basé sur l’Apport médian etabli a partir d’enquetes alimentaires,

les besoins diminue un peu avec l’age et sont moins grand pour les femmes, sauf en cas de grossesse et allaitement. c’est en microgramme et il n’y a pas d’AMT

290
Q

Décrivez a voix haute le metabolisme de l’Iode

A

L’hypothalamus sécrete la TRH, qui fait sécréter la TSH par l’adénohypophyse.

En se liant a son recepteur sur le follicule thyrodiien, ca active le transporteur NIS qui fait entrer l’iode.

Celui ci va se lier a un precurseur de la t4, la thyroxine. L’Iode favorise la sortie du folicule. 2 enzymes sont necessaires a cela, la DUOX1/2 et la TPO.

98% des hormones thyroidiennes sont de la t4. qui sera ensuite activée en t3 pour faire son action.

291
Q

IODE : anref

quel est l’indicateur pour l’Iode?
Comment évolue les besoins selon l’Age et le sexe?
Comment a ton établi l’AMT?
QUel est l’ordre de grandeur

A

REnouvellement et accumulation iode dans la thyroide

CA a donné un ANR

cest pareil homme femme et pareil selon l’age

ca augmente pour grossesse et encore plus pour allaittement.

L’AMT est déterminé par une disfonction thyroidienne.

on parle en centaine de microgrammes par jour.

292
Q

Quelles sont les principales sources alimentaires d’iode?

A

Le Sel fortifié,

Le lait, les fruits de mer et les algues. Ces dernieres peuvent causer l’AMT, on le voit dans certains cas au japon.

293
Q

QUels sont les symptomes associés a la carence en iode?

A
-­‐GOÎTRE
(1ère
manifestaKon)
-­‐HYPOTHYROÏDIE
-­‐RETARD
MENTAL
-­‐CRÉTINISME
(car
les
hormones
thyroïdiennes
sont
impliquées
dans
la
myélinaKon)
294
Q

Quelles sont les consequences lorsque la femme enceinte est en carence en iode?

A

Crétinisme

SYMPTÔMES:
NANISME
RETARD
MENTAL
SURDITÉ
HYPOTHYROÏDIE
TROUBLES
NEUROMUSCULAIRES
La première cause de désordres neurologiques évitables dans le monde
295
Q

DAns quel pays a ton trop ou pas assez d’iode dans l,alimentation?

A

En general, bresil et australie en en trop, russie et asie en on pas assez

296
Q

Comment la déficience en iode cause le goitre?

COmment l’Exces d’iode cause le goitre?

A

En cas de deficience, on fait moins de T4, il n’y a pas de retrocontrole negativ sur l’hypothalamus et l’hypophyse et ça fait qu’on fait trop de TSH, qui stimul etrop la glande thyroide, ca fait l’hypertrophie de la thyroide.

En cas d’exces, on ne sait pas.

297
Q

Quel est le role principal du mangansese?

A

Le manganese est un coafacteur dans la synthese du sulfate de chondroitine, qui est un composant important du proteoglycan du cartilage osseux.

298
Q

Quels sont les 3 autres roles importants du manganese?

A

COFACTEUR DANS lE MÉTABOLISME
DES LIPIDES Synthèse du cholestérol

COFACTEUR DANS LE MÉTABOLISME
DES GLUCIDES
Co-­‐facteur du pyruvate carboxylase

COFACTEUR DE LA SUPEROXYDE DISMUTASE
(SOD)

299
Q

Décrivez les ANREF pour le manganese

Indicateur

variation besoins selon sexe age et cycle de vie

indicateur pour l’AMT

A

C’est un AS

L’indicateur est les apports medieans les plkus élevés liés a aucun signe de carence,

les hommes absorbenet moins et ont donc un AS plus élevé

l’AMT est fixé via un manganese plasmatique élevé associé a une neurotoxicité (parkinson)

300
Q

Décrivez la fonction et le metabolisme du fluor

A

Le fluor vient surtout de l’eau fluorée, il est absorbé tres rapidement. Il se fixe sur les cristaux de minéraux de l’hydroxyapathite, et un peu sur la thyroide. ce qui augmente la stabilité de ces cristaux.

Les enfants absorbent plus et excretent moins que les adultes.

on peut aussi excreter du fluor dans la salive.

301
Q

Quelles sont les autres sources de fluor autre que l’eau?

LEs quelles sont negligeablees?

A
THÉ (FEUILLES)
POISSON AVEC LES OS (EX: SARDINES, SAUMON)
VIANDES MÉCANIQUEMENT DÉSOSSÉES
PRODUITS D’HYGIÈNE DENTAIRE
(DENTIFRICE,
RINCE-­‐BOUCHE) 

rince bouche negligeable car non avalé, meme chose pour dentifrice.

302
Q

Décrivez les ANREF pour le fluor.

A

C’Est un AS, fixé pour la prevention de la carie. C’est un peu pluys haut pour les hommes que les femmes.

La fluorose dentaire a permis de fixer l’AMT.

303
Q

Comment les apports des canadiens en fluor se comparent aux anref?T

A

En general plus bas (un peu,) pour ceux dont l’eua est fluorée mais on considere que c’Est correct. Ce serait trop bas pour ceux sans eau fluorée.

304
Q

Qu’est-ce que la fluorose dentaire?

A

Causé par un exces de fluor dans le développement preeruptif des dents, causent des cernes bruns mais cest surtout esthetique.

305
Q

Le fluor peut il etre toxique pour les adultes et si oui comment?

A

Oui, ca peut augmente la fragilite osseuse, causer des douleurs articulaires du a la calcification des ligaments, mais ça doit etre au dessus de l’AMT chroniquement.

306
Q

Quelles sont les principales sources animales de zinc et son leurs absoprtions en porucentage par le sytsteme digestif?

A
Sources:
Animale:
Foie
Huitres
Viandes, Volailles
Lait
OEufs
307
Q

Quelles sont les principales sources végétales de zinc et son leurs absoprtions en porucentage par le sytsteme digestif?

A

Végétale:
Noix
Céréales
Légumineuses

308
Q

Quel macroélément est ubiquitaire?

A

le zinc

309
Q

Vrai ou faux, les carences en zinc viennent d’un manque dans l’alimentation majoritairement

A

faux, viennet d’autres causes

310
Q

Soit les facteurs suivants, mentionnés s’ils aident ou nuisent a l’Absoprtion du zinc

supplément fer
Allaitement
pHytates
protéines animales (MET HYS CYS)
Age
Citrate
A
supplément fer diminue
Allaitement augmente
pHytates diminue
protéines animales (MET HYS CYS)  augmente
Age diminue
Citrate augmente
311
Q

A la membrane apicale de l,enterocyte, décrivez les échanges en zinc. (entrée, sortie, etc.)

A

ZIP4, absorbtion
transporteur zip 4 Reconnait les 2 sources de zinc. au dela d’une certaine dose ca va rentrer par diffussion passive.

zinc beaucoup sécrétés, plus on ingere de zinc, plus on en secrete. le crops n’a pas intéret a se charger en zinc. car si ça s’accumulait ça causerait des dommages. Donc les secretions digestives contiennent du zinc. plus on en consomme plus on en secrete. facon que le crops a de ne pas accumuler trop de zinc.

312
Q

Comment l’enterocyte stock le zinc? Pourquoi?

A

metallothinéine. mecanisme de defense. elle vient séquestrer le zinc. cest pour maitenir les mineraux a cette proteine et qu’ils s’accumulent de maniere toxique. cette proteine est pas juste dans l,intestin, il y en a aussi dans les autres tissus.

313
Q

Vrai ou faux, la metallotionéine est spécifique au zinc

A

faux

314
Q

Quel transporteur fait passer le zinc dans la circulation sanguine

A

znt1

315
Q

Comment le zinc est-il excrété par le corps? (4 facon)

A

Renal, fecal, integumental, lactational

316
Q

Vrai ou faux, le zinc exerce des fonctions dans presque tout le corps

A

vrai

317
Q

Quels sont les 3 types de fonctions du zinc?

A

catalytique, structurale, régulatrice

318
Q

Nommez les principales enzymes auquel le zinc se lie

A

la Cu-Zn Super oxyde dismutase

La Metallothionéine

319
Q

Comme le zinc agit il sur la métallothionéine et la SOD?

A

EN se liant, l’active

320
Q

Décrivez la fonction structurale du zinc. Aidez-vous de l’exemple de la vitaime D

A

“doigts de zinc” le zinc permet a des recepteurs de se lier a l’ADN. Exemple la zinc se lie au VDR et RXR de la vitamine D et leur permettre de se lier a L’ADN

321
Q

DÉcrivez la fonction régulatrice du zinc

A

COmposant d’un facteur de transcription. Par exemple le zinc s’auto en etant un facteur de transcriptioon pour la metallothionéine. (on l’appele le MTF-1, metal response element binding transcription factor 1.)

322
Q

Vrai ou faux, le zinc serique est un bon marqueur nutritionnel

A

faux mais cest le seul quon a.

323
Q

Expliquer pourquoi le zinc n’est pas un bon marqueur nutritionnel en expliquant la variation des flux de zinc entre le sang et les tissus

A

Plusieurs facteurs font varier le zinc séric.
+ de zinc dans le sang veut dire + zinc va vers les tissus.

Inflammation fait aussi aller le zinc vers les tissus

Les repas favorise le mouvement du zinc vers les tissus.

dun autre coté, le jeune augmente le mouvement des tissus vers le sang du zinc a cause du catabolisme musculaire.

324
Q

Quel indicateur a été choisi pour établir les besoins en zinc?

A

Comber les pertes endogenes.

325
Q

Comment les ANREF du zinc varie selon l’age, le sexe et le cycle de vie, les régimes?

A

Plus élevé pour homme que femme (probablement du a metabolisme plus eleve car plus muscle).

CHez les femmes ca diminue de la puberté a l’age adulte, mais plus gros en grossesse et encore plus gros a l’allaitement, mais pas des changements drastiques.

Augmente x 1.5 avec le vegetarisme du a l’Absoprtion plus faible.

326
Q

Pour les besoins en zinc, quelle est la situation particuliere a surveiller? Comment cela influence les besoins?

A

PErsonnes agées, apports plus faibiles, interactions medicamenteuse, présence de pathologies qui peuvent alterer L’utilisation du zinc, immunité plkus faible.

327
Q

Quelles sont les 2 causes générales de la carence en zinc et les sous causes associées a chacune?

A
  1. apports et absoprtion , du aux faibles apports ou bien syndromes de malabsorption (crohn, coealiaque, fibrose kystique)
  2. Excrétion trop grande, alcoolisme, diabete, HIV (diarrhée)
328
Q

Quel sera la caractéristique principal pour des personnes en carence de zinc depuis qu’ils sont tout jeune?

A

En retard de croissance, tres petit pour leur age

ils ont des carences en zinc. maladie genetique. défaut d’absoprtion au niveau du transporteur intestinal. Carence genetique, probleme de retard de croissance.

329
Q

Quels sont les 4 signes les plus caractéristiques de la carence en zinc? de quels autres signes cette carence peut elle aussi etre accompagnée?

A

Susceptibilité
accrue
aux infections

Lésions
Cutanées

Anorexie

Retard de
croissance

Parfois accompagnés de :
Hypogueusie (↓ goût)
Oligospermie (↓ # spermatozoïdes)
Hypogonadisme
Alopécie
330
Q

Chez les nouveaux nés, quel est le signe principal de carence en zinc? Combien de temps une supplementation en zinc peut avoir un effet?

A

Acrodermatose entéropathique (lesions cutanées du a un defaut d’absoprtion genetique) rapidement, moins de 5 jours

331
Q

Comment a t on établi l’AMT pour le zinc?

A

Détérioration de l’état nutritionnel en cuivre (le zinc chelaterait le cuivre)

332
Q

Le fer hemique a’il besoin d’etre reduit pour entrer dans l’enterocyte?

A

Non, est endocyté sous forme d’heme

333
Q

Quelle est la charge electrique du fer non hemique?

A

3+

334
Q

L’oeuf contient du fer non hemique ou hemique?

A

non hemique

335
Q

Vrai ou faux, le fer non hemique est toujours lié a un autre complexe

A

vrai

336
Q

Sous quelle forme le fer sera il pour etre cofacteur d’enzymes du cycle de krebs? ou pour la replication et la reparation de l’ADN?

A

Fe2+

337
Q

Sous quelle form le fer est il entreposé?

A

Fer ferrique

338
Q
Cytochromes
Catalase
Peroxydase
Myeloperoxydase
Cyclooxygénase

quel nutriment macroelement est impliqué comme cofacteur enzymatique de ces enzymes?

A

Fer ferreux

339
Q

Décrivez a voix haute l’absoprtion et le transport du fer

A

En premier lieu, le fer non hemique (sous forme Fe3+) est reduit sous forme Fe2+ par la protéine Dcytb membranaire apical de l’enterocyte.

Il est ensuite absorbé via le transporteur DMT1 (qui accepte seulement les ions divalents (Divalent metal transporteur)

Dans l’enterocyte, il est stocké par la ferritine, qui peut aussi se trouver dans le sang ou les autres tissus, elle constitue une reserve de fer.

La ferroportine est le canal qui permettra au fer d’aller dans la circulation. La ferroportine est couplée a l’Hephaestine, qui une oxydase cuivre dépendante membranaire basolatérale et qui va oxydé le fe2+ en fe3+ (ferrique). Le ferrique se lie a l’apotransferrine, qui devient transferrine une fois dans le sang. elle est la principale proteine de transport du fer

D’un autre coté, l’heme se lie directement a un recepteur specifique (heme receptor) sur la muqueuse intestinale. Il est endocyté puis digéré par l’heme oxygenase, qui libere un ion ferreux, qui va pouvoir lié la ferritine ou aller a la ferroportine.

340
Q

Décrivez a voie haute les échanges de fer entre le sang et les differents tissus pertinents

A

Le fer est ingéré puis excrété par le tube digestif. La transferrine echange du fer avec le foie via la ferritine qui le stock.

Du fer plasmatique va aussi a la moelle osseuse, qui fait des nouveaux globules rouges qui ont besoin d’Heme et donc de fer.

Certains globules rouges lyse et l’heme va au foie pour etre reéutilisé ou sécrété.

sinon, les globules rouges en fin de vie sont recyclés au plasma sanguin via le systeme reticuloendothelial (RES).

341
Q

Que peut on dire du recyclage du fer en quelques mots?

A

generalement bien recyclés

342
Q

Quelles sont les 4 principales voie d’excretion du fer?

A

Sang, Feces, Peau/cheveux et poils , Urine

343
Q

QUelles sont les principales perte de sang chez lhomme en lien avec l’excretion du fer?

A

DOn de sang ou sang menstruel

344
Q

Quels facteurs influencent la perte de fer via les menstruations?

A

Thérapie
Remplacement
hormonal augmente les pertes

Contraceptifs
Ménopause
DIminue les pertes

345
Q

Quels facteurs augmentent la perte de fer via les celles?

A

Exercice vigoureux, Sang occulte
Desquamation
Sécrétions

346
Q

Pourquoi l’Absoption du fer est tres variable selon l’endroit du monde ou l’on se trouve?

A
ex , amerique du nord
20-25%
\+++Viandes, volailles, poissons
Acide ascorbique
Régime diversifié

vs afrique:

5-10%
+++Céréales
Moins de viandes, volailles, poissons
Plus de Phytates

347
Q

Quels sont les 4 principaux facteurs qui affectent la biodisponibilité du fer et comment leur variation augmente ou diminue celle ci?

A

Type de fer, le hemique est mieux que le non hemique

BEsoin en fer, plus on en a besoin plus on consomme

Plus la dose est grande plus faible est l’absorption

Facteurs nutritionnels:
acide ascorbie augmente, proteines animales augmente,
phytates, calcium, café, thé, polyphenols, proteines vegetales, diminuent

348
Q

Besoin en fer et biodisponibilité

Décrivez la variation du pourcentage d’absorption selon les réserves en fer pour les 4 situations suivantes:

Fer hemique

fer non hemique

fer non hemique + 25 mg acide ascorbique ou 30 g de viande

fer non hemique + 75 mg acide ascorbique ou 90 g de viande

A

Pourcentage passe de 35% a 0 mg de reserve et diminue jusqua 15% pour 1000 mg de reserve.

Pourcentage passe de 5% a 0 mg de reserve et diminue jusqua 2% pour 1000 mg de reserve.

Pourcentage double par rapport a non hemique

Pourcentage double encore (donc 4x l’initial) par rapport a non hemique

349
Q

Comment évolue les besoins en fer selon l’Age, le sexe et le cycle de vie?

A

Pour les hommes, c’Est moins que les femmes et ca diminue avec l’age

LEs femmes cest plus élevés, ca augmente a partir de 19 ans, mais diminue a partir de 51 ans comme on est ménopausé.

en grossesse ca augmente bcp car plus de volume sanguin. ca diminue ensuite durant l’allaitement car pas de menstruation, un peu comme femme menopausé.

350
Q

Fer ANR: situations particulières

Décrivez les situations pârticulieres et comment cela impact les besoins. ( 4 situations)

A

femmes sa diminue ça si on est sous contraceptifs oraux

FEmme ca reste élevé si on est ménopausé mais sous therapie de remplacement hormonal car on a jtoujours les regles.

Exercice intense augmente de 1.3 a 1,7 x car ça augmente les pertes (hemolyse des globules rouges), donc on en amene plus au corps.

Vegetarisme augmente aussi les besoins de 1.7 -1.8 x car fer non hemique moiins biodisponible.

351
Q

Vrai ou faux, les vegetaliens absorbent moins bien le fer que les vegetariens

A

vrai

352
Q

Fer

Pourquoi le végétarisme augmente tant les besoins?

A

1) Peu ou pas de protéines animales qui ont un effet promoteur
sur l’absorption du fer
2) Forme non-hémique est moins biodisponible

353
Q

Décrivez les 3 stades d’anémie ferriprive (a voix haute)

A

stade 1, reserves en fer diminue, ferritine serique diminue, fer serique diminue, aucun signe physique

Stade 2, diminution du fer disponible pour Hb, pourcentage de saturation de la transferrine diminue, augmente le recepteur a transferrrine, pas de signe physique

Stade 3,
Diminution de l’erythropoiese, l’Hg diminue, l’hematocrite diminue, le volume globulaire diminue, signes physiques : paleurs, fatigue, intolerance a l’exercice, plus susceptible aux infections, retards neurodevelopmentaux chez l’enfant, pica (mange des trucs bizarres), problemes de thermoregulation

354
Q

Quelles sont les 4 grandes causes de l’anémie ferriprive? Et leurs sous causes respectives. (a voix haute)

A
Apports/Absorption 
Faibles apports
Apports Fer peu biodisponible
Syndromes de malabsorption
Infection H.Pylori
Besoins 
Croissance 
Grossesse
Grossesses multiples
Règles abondantes

Pertes
Athlètes d’endurance
Parasitose Intestinale (pays en developpement)
Sang occulte (test de Guaiac) (test du depistage des celles dans le sang)

Pathologie
Insuffisance rénale chronique

355
Q

Fer, pouruqoi Insuffisance rénale chronique cause l’anemie ferriprive?

A

l’EPO, stimule la synthese des globules rouges, produites par les reins. on fait pas d’EPO, et on fait pas les globules rouges. carence de produire des globules rouges.

356
Q

Quelle est la prevalence de l’anemie ferriprive dans le monde, quel sous groupe de la population touche elle le plus?

A

25%

surtout femmes en age de procréer, enfants 12-36 mois et pays en développement

357
Q

Pourquoi les enfants d’age prescolaires sont plus a risque d’anemie ferriprive?

A

Pourquoi enfant age prescolaires? vont prendre du lait de vache, lait de vache nest pas une bonne source de fer. a lage ou on commence a manger du solide, on commence a vivre sur les reserves, et elles se vident, surtout si lenfant mange pas de viande et boit du lait de vache qui na pas beaucoup de fer. Cest important pour le développement du cerveau donc a surveiller.

bébés naissent avec reserves de fer, maintenues, car lait maternel contient un peu de fer, le bebe peut vivre avec ça.

autour de 6 mois, on doit introduire tout (meme la viande par exemple), pour donner du fer.

358
Q

Quelles sont les consequences de l’Exces de fer?

en quel 3 cas est-ce observé?

Comment l’AMT a-t-il été fixé? quel source concernte til?

A

Constipation Selles noires
Diminution absorption zinc
Hémosidérose (bcp fer foie)

Transfusions sanguines excessives
Prise d’une supplémentation massive et prolongée
Absorption excessive

AMT: Troubles gastro-intestinaux, pour fer non hemique en supplements

359
Q

Fer, Décrivez l’hemochromatose.
la cause et ce qui en decoule.

L’apparition des symptomes avec l’age

le traitement et l’Esperance de vie

A
  1. Maladie génétique
  2. Augmentation absorption intestinale fer
  3. Accumulation fer dans organes/tissus
on commence par etre asymptomatique, puis vers 20 ans on a des symptomes non specifique, pour finalement comment a avoir du dommage aux organes vers 30 ans, diabete vers 45 ans, mort prematurée. 
Foie Cirrhose foie Cancer foie 
Coeur  Cardiopathie 
Peau  Pigmentation  
Pancréas Diabète

traitement: saignée thérapeuthique.
autre traitement: on donne des chelateurs qui vont lier le fer. empeche de s’accumuler. c’Est une maladie qui se traite bien, ne cause plus de morts précoces mais doit etre traitée a vie.
A la naissance on test les bébés pour une serie de maladie genetique.

360
Q

Pourquoi l’Exces de fer est risqué d’un point de vue biochimique?

A

Risques d’un excès de fer: effet prooxydant
Le fe3+ libre ou complexé dans le sang augmente, devient rapidement du fe2+, puis participe a des reactions qui generent des radicaux libres, ce qui fait le cancer et les MCV

361
Q

La rédaction d’une prescription

Dans quel cas un nutritionniste peut prescrire, et que peut il prescrire?

A

Quand une ordonnance medicale indique que la nutrition constitue un facteur determinant du traitement de maladie,

vitamines et mineraux
materiel dalimentation enterale necessaire
solution denzymes pancreatiques servant a retablir la fonctionnalité du tube d’alimentaton

362
Q

Que doit contenir la prescription??

information sur le patient

Ordonnance

informations du nutritionniste

A
Informations sur le patient, • Nom du patient
• Date de naissance ou # RAMQ
• Date de rédaction de l’ordonnance
• Période de validité de l’ordonnance,
lorsque justifié par la condition
du patient
• Toute contre-indication ou
renseignement pertinent, le cas échéant
Ordonnance
• Nom intégral du médicament
• Posologie
• Voie d’administration
• Durée du traitement ou
quantité prescrite
• Nom d’un médicament dont le patient
doit cesser l’usage si pertinent
• Interdiction de procéder à
une substitution de médicament
lorsque la situation le requiert
• Votre nom
• Votre numéro de permis d’exercice ainsi
que votre numéro de prescripteur
• Le nom et les coordonnées
de l’établissement où vous travaillez
• Votre signature
363
Q

Quels sont les 2 types de supplements de fer?

A

–Inorganique:

organique

364
Q

Nommer les supplements de fer organique et inorganique et indiquez celui qui est le mieux absorbés

A
–Inorganique:
• Sulfate ferreux
–Organique:
• Fumarate ferreux
• Gluconate ferreux
• Bisglycinate ferreux serait mieux tolérés que les autres (gluconate ou fumarate moins bien tolérés).
365
Q

Combien de fer un supplement de 300 mg va compter?

A

Depend de la molecule. le fer elementaire est generalement 1/3 ou moins du 300 mg total par pilule de supplement.

366
Q

Vrai ou Faux? Il faut faire attention à ne pas dépasser l’AMT du fer
lorsqu’on prescrit un tel supplément
pour corriger une carence.

A

faux:
l’AMT concernent les gens en bonne santé. quand on corrige une carence nutritionelle, on est en situation clinique, l’AMT cest pour une situation normale. C’est donc permis quand on corrige une carence. les risques liés au supplements seraient dans un cas normal.

367
Q

Quels sont les formes de suppléments en fer quet qu’est-ce qui motivera la décision de prescrire une forme ou l’autre?

A

Types de suppléments
• Voie orale (per os): comprimés ou sirop
• Supplément injectable (IV)
valider si la carence est liée a un trouble de l’absoprtion. on pourrait avoir besoin d’y aller par le injectable au lieu de voie orable.

aussi pour la tolerance. par exemple une personne agée pourrait setouffer avec une grosse pipule.

368
Q

Comment choisit on la dose a prescrire pour un supplement de fer??

A
  • Dépend de votre évaluation du statut nutritionnel en fer et de l’alimentation
  • Si carence: dose prescrite peut dépasser l’AMT.

depend des valeurs biochimiques et de l’alimentation de la personne.
a
quelquun qui a un statut tres carencé va nécéssiter des doses plus élevées.

369
Q

Ordonnance Fer : Fréquence d’administration , décrivez les 4 points importants

A

• Il est préférable de diviser en 2-3 doses quotidiennes pour aider la tolérance
• À prendre préférablement avec ou 30 minutes avant les repas (tout dépend
de la composition du repas et tolérance)
• Éviter aliments riches en calcium et en caféine lors de l’ingestion du
supplément
• Prise avec jus d’orange (ou autre source de vitamine C) peut être
préconisée

370
Q

Ordonnance Fer : Pendant combien de temps?

A

Réponse peut-être rapide (qqs semaines) mais réévaluer le statut après 3
mois de supplémentation et prolonger la supplémentation si nécessaire

371
Q

Ordonnance Fer : Effets secondaires possibles , quels sont ils? (6)

ces effets sont ils seulement si la dose est superieure a l’AMT?

A

Symptômes gastro-intestinaux sont les plus couramment rapportés:
§ Surtout constipation
§ Selles noires
§ Goût métallique
§ Nausée/vomissement
§ Diminution appétit
Autres effets possibles: Taches brunes (attention chez les enfants: dents)

Ces symptômes sont liés à la prise du supplément et peuvent survenir même avec des
doses < AMT

372
Q

Vrai ou faux, le dietetiste doit verifier les interactions medicamenteuses avant de prescrire un supplement

A

Vrai

373
Q

Quels medicaments diminuent l’absopriton du fer?? Comment peut on contrer cet effet si un patient a besoin dun supplement

A

Penicillamine Imunosupressuer
Tétracyclines (antibiotiques) et dérivés C
iprofloxacin et dérivés (quinolones) (antibio)
Levodopa/carbidopa (parkinson)
Captopril (hypertension)
prendre 2 heures decart avec le supplement

Antiacides (eviter ou prendre 2 heures decarrt)

374
Q

Quels sont les contreindications pour le supplement de fer? (4)

A

Hémochromatose
Hémosidérose
Transfusions sanguines régulières
Anémie hémolytique

375
Q

Pourquoi l’Anemie hemolytique est une condition qui contre indique la supplementation en fer?%

A

les globules rouges shemolysent facilement. le fer n’est pas necessairement perdu. il est recupéré par le corps. les globules rouges sont lysés (souvent genetique), fer contenu est recyclé. ces gens n’ont donc pas besoin de fer. le medecin va avoir “flaggé” ça, on ne verra probablement pas ça.

376
Q

Soit la prescription suivante, que veut dire PO et BID?

Diagnostic d’anémie ferriprive par médecin de famille
45 mg de fer sous forme de sulfate ferreux
PO, BID
Au dîner et au souper
Pendant 3 mois, à partir de ce jour

A

Per Os, par la bouche, BID, 2 fois par jour

377
Q

1) Après trois mois de traitement, le statut nutritionnel en fer de Mme Lortie
est encore faible et je juge que je dois prolonger la prescription de sulfate
ferreux.
Ai-je l’obligation d’en parler au médecin traitant avant de procéder ?

A

pas obligé de demander au médecin. on est pas dans un nouveau diagnostic. on peut renouveler si on juge que c’Est ce qu’il faut faire. mais si on suspecte une autre cause, on a oui l’obligation de demander au médecin.

378
Q

2) Lors de son rendez-vous de suivi, ma patiente me demande si je peux lui
prescrire des suppléments de calcium et de vitamine D pour sauver du temps.
Ai-je le droit d’accéder à sa demande ?

A

pas le droit.
on a pas eu l’ordonnance du medecin pour ça.
on peut juste recommander d’aller acheter le supplement si on juge que c’est nécéssaire.

prescription doit etre liée a son diagnostic. on ne pourrait pas prescrire de la vit D et calcium car le diagnostic et l’ordonnance est pour le fer.

379
Q

3) Diététiste-nutritionniste en pratique privée.
Suite à l’évaluation nutritionnelle d’un patient, elle aimerait lui prescrire un
supplément de vitamines/minéraux mais son patient n’a pas de médecin de
famille et n’est pas suivi par un spécialiste.
Peut-elle quand même procéder à la prescription étant donné les
circonstances?

A

pas de diagnostic, pas d’ordonnance, donc non.

380
Q

4) Est-ce qu’une diététiste-nutritionniste détient le droit de prescrire des
suppléments d’oméga-3 et des probiotiques ?

A

ne peut pas prescrire car pas des vitamines et minéraux.
pas tous les supplements de la terre que l’on peut prescrire, seulement les vitamines et minéraux.

encore ici on pourrait recommander si on jugeait que le la patient a besoin.

381
Q

A voix haute décrivez l’absopriton et l’excretion du phosphore et et les facteurs qui l’augmente ou la diminue.

A

Absopriton se fait via le NaPi2b, transporteur couplé au sodium, l’Absoprtion se fait surtout au niveau proximal.
la vit D augmente l’expression du gene du NaPi2b, et aide l’absoprtion.
LEs phytates et les phosphopeptides-caséine chelate le phosphore et diminue l’Absorption.
les antiacides rendent le phosphore moins solubles et diminuent labsoprtion. La PTH diminue la reabsopriton reinale de phosphore et favorise donc l’excretion, qui se fait majoritairement via l’urine.

382
Q

A voix haute décrivez l’absopriton et l’excretion du magnesium et et les facteurs qui l’augmente ou la diminue.

A

Magnesium est absorbé a 35%, surtout au niveau proximal et distal. LEs transporteurs TRPM6
TRPM7 sont responsables et ne sont pas regulés par la vit D. Le phosphore et le zinc chelatent le magnesium et viennent nuire a l’Absorption. Les phytates chelatent aussi mais generalement vont etre dans des aliments qui ont bcp de magnesium donc ca compense. plus la dose est grande moins l’absopriton est bonne.
La PTH augmente aussi l’Excretion via l’urine.

383
Q

Quelles sont les fonctions et la repartition du phosphore dans le corps

A

la majorité sert pour l’os (80%)

cest aussi tres important dans le muscle squelettique (9%) puis le sang et les tissus (11%) ou il sert a la phosphorilation des proteines, ATP, creatine P, phospholipides, Compose aussi les vit, ADN, ARN, enxyzymes.

384
Q

Quelles est la fonctions et la repartition du magnesium dans le corps

A

60% dans l’os, 30% muscle squelletique, 10$ dans les tissus, 1% sang

Cofacteur
enzymatique
de + 600
réactions
enzymatiques
385
Q

Quelles sont les 3 principales fonctions du magnesium?

A
Co-facteur
enzymatique 
Ex: enzymes
impliquées
dans 1) synthèse PTH,
2) Synthèse Glutathion
3) Cycle Krebs
Fluidité
Perméabilité
membranaire
Affecte fonctions
transporteurs
Échanges cellulaires
Canaux ioniques
Régulateur
Échanges ioniques
(canaux)
Maintien
Concentrations
Intracellulaires
Ca et K
Antagoniste Ca
386
Q

Pourquoi pense on que le magnesium peut prevenir les crampes?

A
Car  
Régulateur
Échanges ioniques
(canaux) 
vient de là, les crampes pourraient etre en lien avec le calcium, on pense que le fait de donner du Mg en antagonisant le Ca va venir alléger la contraction.  Cest la que le magnesium est étudié en lien avec les crampes musculaires.
387
Q

Comment evolue les besoins en phosphore selon lage, puis pour le sexe et le cycle de vie? Quel est le critere retenu?

A

Enfants, cest plus bas que a a l’adolescence (car croissance, formation des os) puis ca diminue de 19 ans jusqua la mort. PAs de difference pour femme enceinte ou allaitement. pas de difference pour le sexe
critere: maintien phosphorémie

388
Q

Comment evolue les besoins en magnesium selon lage, puis pour le sexe et le cycle de vie? Quel est le critere retenu?

A

Bas pour enfants, puis augmente significativement pour ados, rediminue avec lage pour les femmes mais redevient plus grand a partir de 50 ans. Pour les hommes , ca augmente avec la puberté, rediminue a l’Age adulete puis réaugmente apres 50 ans.

les hommes ont besoin de plus que les femmes,

le critere retenu est la balance de mg a l’Equilibre.

389
Q

Comment l’AMT du phosphore a été établi et pourquoi? COmment évolue t il avec l’Age?

A

Calcification des reins, (acidité - risque de calculs reinaux), ca augmente de l’age enfant a adulte mais rediminue apres 70 ans car le rein est moins fort

390
Q

Comment l’AMT du MG a été établi et pourquoi? COmment évolue t il avec l’Age?

A

AUgmente avec l’Age , les troubles GI sont le critere pour letablir.

391
Q

Décrivez a voix haute l’arbre decisionnel pour dépister la carence en magnesium

A

On regarde la magneseme, si elle est basse, on fait un test d’urine 24 h de Mg, si cest bas, il y a déficience, si cest trop haut, on a une deficience du aux reins qui en jettent trop.

SI la magnesemie etait ok, on fait aussi le test urine 24h. Si cest normal, pas de deficience. Si cest bas, on fait un Mag Load test et on regarde le magnesium dans l’urine. Si on retient peu de magnesium (urine bcp), on est correct, le corps en a pas besoin, si on urine peu, (beaucoup de retention), on a une carence, le corps en a besoin.

392
Q

Quels sont les 2 principales causes de deficiences en phosphore?? Décrivez comment les 2 causes peuvent mener a la phosphaturie

A

Alcalose respiratoire Hyperventilation excrete bcp de CO2, ca diminue le CO2 intracellulaire et du sang, augmente le pH intracellulaire, et augmente la phosphorylation du glucose et autres reactions de phosphorylation. Donc le phosphore quitte le sang pour aller vers les cellules ce qui cause le bas taux de phosphore.

Déficience causée
par un problème d’apports/
absorption/métabolisme
Alcoolisme chronique
Hyperparathyroïdisme (augmente PTH donc phosphaturie)
Déficience en vit.D (diminue absopriton intestinale)

393
Q

Déficience (magnésium)

Quelles sont les 3 grandes causes? et les sous causes?

A

enterocyte peuvent trop sécréter (Diarrhées
Vomissements ) ou pas assez absorber. (Syndromes
Malabsorption Résection intestinale )

Reins peuvent faire des pertes excessives (diurétiques ou diabete)

394
Q

Expliquez Le cercle vicieux du magnésium et diabète

A

LE diabete cause la diurese osmotique (urine riche en glucose, mg, (autres osmolites). perte urinaire accrue
ce qui augmente les pertes urinaires, notamment en Mg, cause la carence en Mg, mais le mg est un cofacteur de la secretion et aussi impliqué dans laction de l’insuline. donc cercle vicieux car ca empire le diabete, qui empire la diurese … etc.
donc gens diabetique sont tres surveillé pour le statut en mg.

395
Q

Manifestations de déficience en magnésium.

Décrivez les manifestations et remontez jusqua la cause.

A
Tétanie
Signes Chvostek et Trousseau
Spasme
sont des manifestation neuromusculaires
causés par Hypocalcémie
car la carence en Mg diminue la secretion de PTH, et cause la resistance a la Vit D et a la PTH.  

Aussi, le mg a des effets sur les echanges ionique, si deficient, ca augmente les pertes en potassium, ce qui diminue le K intracellulaire, fait l’hypokaliémie, puis l’arythmie cardiaque

396
Q

Décrivez le signe de trousseau

A

est un spasme causé par un probleme au niveau neuro musculaire. on le diagnostic en serrant le bras et la personne va réagir en ayant un spasme au niveau de la main. Mais ça pourrait aussi etre un signe de carence en calcium.

397
Q

Décrivez le signe de chvostek

A

spasme, suite a avoir tapoté la joue, on va le voir en nutrition clinique,

398
Q

Quelles sont les différentes formes de calcium et ou se trouvent elles??

A

Calcifié ectopiquement, dans els arteres ou reins, c’est pathologique, lorsque la concentration de Ca et P depasse le point de saturation, souvent associé a maladie de rein.

Calcium complexé dans l’os (99%)

Calcium sanguin (0.1%), soit lie a lalbumine ou globuline (40-45%), soit ionisé (45-50%), soit lié en complexes (8-10%)

Dans les tissus (0.9%) il est séquestré

399
Q

Quelles sont les fonctions du calcium dans l’Os?

A

Cristaux d’hydroxyapatite, 40% du mineral osseux est du calcium, l’os est la reserve corporelles de calcium, et participe au maintien de la calcémie.

400
Q

Quelles sont les fonctions du calcium dans la dent?

A
Cristaux d’hydroxyapatite
\+ denses que ceux de l’os
Contient + fluor: stabilise l’émail
Le rend moins mobilisable
Ne participent pas au maintien
de la calcémie
401
Q

A voix haute, Expliquer la fonction de second messager du calcium

A

LEs stimulis favorisent l’entrée de calcium via des canaux calciques ou bien la mobilisation du Ca du reticulum endoplasmique. Le Ca cytosolique augmente ce qui active des protéines et permet la réalisation de fonctions biologiques.

Par exemple lorsque le spermatozoide entre dans l’ovule cest le stimuli qui favorise l’entrée de calcium et permet la premiere division cellulaire.

autre exemple, glucose reconnu par la cellule beta, ca augmente le ca2+, qui active proteines et va augmente la secretion d’insuline

402
Q

En general, quel sous groupe de la population a une balance en calcium favorable ? Pouruqoi?

A

LEs personnes de races noire, serait peut etre parce que les niveaux de vit D sont naturellement plus bas.

403
Q

Entre les femmes et les adolescentes, lesquelles ont une balance en calcium la plus favorable?

A

Adolescentes

404
Q

Décrivez les mouvements du calcium dans le corps

A

Absorbé dans l,entecocyte, mais aussi sécrété dans l’enterocyte par secretions digestives ou desquamations, puis exrection fecale.
Une fois dans le sang, il y aura soit exreciton cutanées (sueur peau, cheveux) faible fraction), ou absorption puis /ou résoprtion osseuse. ou finalement excretion urinaire.

405
Q

Chez une femme adulte, quelle sera la balance entre absorption et resoprtion osseuse du calcium?

A

a peu pres egal.

406
Q

Comparez l’excretion du calcium pour les femmes adultes, adolescentes blanches et adolescentes noires

A

Adolescentes noires excrete moins, ensuite adolescentes blanches, ensuite femmes adultes

407
Q

Décrivez a voix haute l’Absoprtion du calcium

A

Le calcium va dans le sens du gradient par un canal calcicique vers l’enterocyte. La saturation de ce transportreur arrive avec des doses au dessus de 500 mg. Du calcium traverse aussi par voie paracellulaire lorsque les doses sont au dessus de 120 mg.

Dans la cellule, le calcium se lie a la D9K (la calbinding protein) qui le transporte dans le cytoplasme. Il est ensuite secreté dans le sang via une pompe PMCA1b qui a besoin d’ATP pour faire passer le Ca2+ dans le sang, contre le gradient. Secondairement, un echangeur sodium calcium peut aussi faire entrer du calcium dans le sang.

l’ABSOPRTION se fait surtout dans le jejunum et l’iléon

le ecac, calbinding protein et PMCA1b sont toutes vitamines D dépendantes (up-régulée par la vit D).

408
Q

Soit les facteurs suivants, dites leur influence sur l’Absoprtion de calcium et pourquoi

Statut vit D

augmentation Transit intestinal
Diminution muqueuse intestinal

Déficience phosphore
Exces phosphore

Diminution acidite gastrique( ex antiacides)

Stades de la vie

Facteurs alimentaires, doses

A

Statut vit D (augmente ou diminue avec le statut)

augmentation Transit intestinal
cas de diarhée, va baisser le calcium. quand ca passe vite on absorbe pas bien dont le calcium.

Diminution muqueuse intestinal
dimue dans les cas de resection intestinale.

Déficience phosphore
carencé en phosphore va faire plus de vit D active, ca va audonc augmenté l,absoprtion de calcium.

Exces phosphore
diminue labsorption de calcium via diminution vit D (active)

Diminution acidite gastrique( ex antiacides)
va diminuer l’absoprtion car dissout moins bien le calcium. calcium moins bien dissout est moins bien absorbé.

Stades de la vie
peut augmenter ou baisser. (augmente ou diminue

Facteurs alimentaires, doses (augmente ou diminue

409
Q

Décrivez levolution de l’absoprtion du calcium selon les stades de la vie suivant et pourquoi

nourrissons
enfants
puberté
adultes
Grossesse
Personnes agées, femmes ménoposées
Apres 40 ans
A

nourrissons +++++ (intestin plus permeable au calcium et plus d’espace entre enterocyte
enfants ++
puberté +++ (hormones sexuelles augmentent absoprtion Ca)
adultes +
Grossesse ++++ (donc grossess fait quon secrete plus de pTH on synthetise plus de vit D. )
Personnes agées, femmes ménoposées -

410
Q

Calcium Effets de la dose ou du repas

« Plus on en ingère, ______ on en absorbe »

A

« Plus on en ingère, moins on en absorbe »

411
Q

Calcium Effets de la dose ou du repas

Effet du repas: augmente absorption
Pourquoi?

A

1) Retard vidange gastrique: meilleure dissolution du calcium
2) Ralentissement transit chyme: meilleur contact
avec la muqueuse intestinale, favorise absorption
paracellulaire

412
Q

Placer les sources suivantes dans le graphique Biodisponibilité (en x) et teneur en calcium (en y)

légumes verts feuillus
Produits laitiers
Poisson en canne
Noix
legumineuses
Brocoli, choux de bruxelles, radis
A

légumes verts feuillus (5%, moyenne teneur)
Produits laitiers (30-35%, haute teneur)
Poisson en canne ( 30-35%, teneur un peu en haut de la moyenne)
Noix (20%, teneur un peu en haut de la moyenne)
legumineuses (teneur et biodisponibilité (15%) en bas de la moyenne)
Brocoli, choux de bruxelles, radis 50-60% mais plus faible teneur

413
Q

Pourquoi les produits laitiers sont une bonne source de calcium? 4 facteurs

A

Lactose augmente absoprtion chez nourrisson

Phosphopeptides caséine, favorise meilleure dissolution dans lestomac

Glucose, sucros, pas d’effet, donc lait au chocolat pas un enjeu par exemple

Phosphore lipides, pas d’effet , mais phosphore est un chelateur en exces

414
Q

Décrivez les différents facteurs alimentaires et leurs influences sur l’absoprtion du calcium pour les sources vegetales

A

Fibres emprisonnennt physique et font que moins d’absoprtion

oxalate, surtout dans epinards, etc. ont un effet chelateur, comme les phytates. Par contre la fermentation attenue leffet des phytates, car les bacteries vont produrie des phytases qui digerent les phytates alimentaires. certains produits fermentés dont le calcium va etre plus biodisponible. autres choses intéressantes, germination, trempage, etc. va faire augmenté la biodisponibilité du calcium. des sources vegetales. tous des effets sur les

415
Q

Calcium:
Facteurs alimentaires:
Effet positif des fibres

quelle fibre est benefique sur l’absoprtion du calcium et pourquoi?

A

Fructooligosaccharides
Sont fermentés dans le colon: production d’AGCC, qui abaisse le pH du colon, ce qui augmente la solubilité du calcium pour le faire passer par voie passive, et les AGCC forment aussi des complexes avec le calcium donc ca fait une meilleure absorption.

De plus, ca stimule la proliferation des bacteries qui font les phytases ce qui libere le calcium chelaté par les phytates.

416
Q

Calcium
Facteurs alimentaires: Sodium
Pourquoi une grande consommation de sodium nuit a la balance de calcium???

A

Un apport élevé de sodium: augmente excrétion urinaire de sodium
++ excrétion urinaire de sodium:é++calciurie
on a des transporteurs reinaux commun entre calcium et sadium et ça fait quon excrete aussi du Ca.

417
Q

Calcium
Facteurs alimentaires: caféine
comment le café influence l’absorpétion de calcium et comment on peut mitiger cette effet?
Que faut il aussi prendre en compte au niveua de la cafeine et de la balance en calcium?

A

dimunue absorption de calcium (5mg/tasse de café)

1-2 c. à table lait/breuvage laitier

Mais la caféine augmente aussi les pertes urinaires de calcium

418
Q

COmparez l’alimentation des pays en développement vs développés et les cas d’osteoporoses

Comment explique on cela?

A
Pays industrialisés
850 mg Ca par jour
\++OSTÉOPOROSE
103 g protéines
60 g protéines
animales
Pays en
développement
350 mg Ca par jour
PEU OSTÉOPOROSE
59 g protéines
13.3 g protéines animales

LEs aliments riches en protéines animales (AA soufrés) font une excretion accue des produits soufférs par le rein, mais ces produits complexent le calcium et favoirse les pertes urinaires. De plus, ces prots ont une charge acide nette, et consomment moins de tampons (qui sont plus associés aux prots vegetales) et les os, reins et poumons doivent donc fournir le tampon a la place, ce qui fait la resorption osseuse et mene a l’osteoporose

419
Q

=

A

=

420
Q

Vrai ou faux, le citron a une charge acide nette plus grande que le fromage cottage

A

Faux

421
Q

Facteurs alimentaires: Phosphore
Décrivez comment la consommation des phosphores se produit et impact labsoprtion de calcium, et quel groupe de la population est plus touché par ce phenomene

A

Quand on mange du P, la Phosphoremie augmente, mais aussi ca aumgente la chelation intestinale de calcium et augmente la secretion digestive de calcium

la phosphoremie qui augmente augmente la PTH et Diminue le calcitriol. (vit D active) qui dimune labsoprtion de calcium.

Laugmentation de PTH fait aussi que sa stimule la resoprtion osseuse donc fragilise les os.
EFFET EST BEAUCOUP PLUS MARQUÉ
SI LA CONSOMMATION ÉLEVÉE DE PHOSPHORE
EST COMBINÉE À UN FAIBLE APPORT DE CALCIUM

On remarque ça bcp chez les ado, mangent moins bien et boive bcp de boissons gazeuses qui ont de lacide phosphorique

422
Q

Calcium, Facteurs alimentaires: Lipides

Décrivez comment les pathologies qui diminue la digestion des lipides impactent l’Absoprtion du calcium

A

quand on malabsorbe les lipides, ça saponifie le calcium (savon) et fait que le calcium nest pas absorbé. ex en fibrose kystique on malabsorbe les lipides et aussi on va donc malabsorber le calcium. dans un contexte physiologique

423
Q

Comment les Anrefs évolue selon lage pour le calcium?

A

Augmente avec l’age, puis idminue legerement a l’age adulte, mais reaugmente apres 50 ans pour les femmes et 70 ans pour les hommes.

424
Q

Comment les Anrefs évolue selon le cycle de vie pour le calcium?

A

Si grossesse ou allaitement, cest la meme chose que pour les femmes.

425
Q

Comment les Anrefs change selon le sexe pour le calcium?

A

égales

426
Q

Comment previent on la perte osseuse avec l’age?

A

uest-ce qui empeche la perte de masse osseuse?? les hormones principalement (femme qui prenne des oestrogenens vont prevenir la perte de masse osseuse). combiné a des apports en calcium optimaux plus l’exercice physique est-ce qu’il y a de plus efficaces pour maintenir la masse osseuse. on voit que les medecisn predisent de moins en moins e spplements de calcium, plutot supplements de vit D.

ex, a la menopause, de prendre un supplement de vit D et calcium naidera pas necessairement.

427
Q

Régulation de la calcémie, décrivez la a voix haute.

A

C’est finement régulé, des variations de plus ou moins 10% indiquent une maladie.

Si elle diminue, les recepteurs au calcium des glandes parathyroides stimule la secretion de PTH. Celle ci favorise la resoprtion osseuse et la réabsoprtion tubulaire de calcium aux reins. ce qui dans les 2 cas augmente le calcium.

Le rein stimulé par la PTH stimule la fabrication de la vit D active via une uprégulation de la CYP27B1 (1 alpha hydroxylase) , ce qui va augmenter l’Absoprtion de calcium et aussi la resoprtion osseuse, et aussi augmenter la réabsoprtion tubulaire dans le rein,/

En cas d’Augmentation de la calcémie, les CaRécepteurs de la thyroides stimule la sécrétion de calcitonine, qui va diminuer la réabsoprtion au niveua des reines et va diminuer la resoprtion osseuse.. la calcemie diminue

428
Q

Comment les apports faibles en calcium peuvet augmenter la lipogenense et la pression arterielle?

A

Diminution calcemie augmente la vit D, qui augmente le CaR des cellules musculaires et adipeuses.

le ca intracellualire du muscle favorise la contraction et donc la pression arterielle.

le ca du gras intracellulaire augmnete la lipogenese.

429
Q

Comment l’AMT pour le calcium at il été établi?

A

Nephrolithiases

430
Q

Calcium

Comment l’AMT varie selon l’age?

la grossesse?

l’Allaittement?

le sexe??

A

Enfants ++
9-18 +++
19-50 ++
51-70 + (declin de la fonction reinale)

grossesse 14-18 +++
grossesse 19-50 ++

Allaitement
14-18 ans +++
19-50 ans ++

sexe pareil

431
Q

Lors d’apport excessif en calcium , quels sont les risques?

A

-Hypercalcémie
-Néphrolithiases** (Prédisposition génétique)
-Insuffisance rénale
-Syndrome du lait et des alcalins
-Interactions indésirables avec certains éléments
(Fer, Zinc, Magnésium, Phosphore)

432
Q

Calcium et pierres au rein
de quoi sont fait les nephrolithiases?

Dou viennent les oxalates?

Comment explique on que la diete riche en calcium reduit les pierres au reins? l’inverse est il vrai?

A

surtout faites de cristaux d’oxalate de calcium

L’oxalate peut être de source exogène (aliments) et endogène
(produit par notre corps)

Une diète au contenu élevé en calcium: réduit l’absorption
d’oxalates par chélation intestinale et ↓le risque de
néphrolithiases. oui cest vrai

433
Q

Calcium et santé cardiovasculaire

À la fin des années 2000, une étude a rapporté une _____ risque
d’infarctus du myocarde chez ceux qui prenaient des
suppléments de calcium
§ L’augmentation de la ______ vasculaire était le
mécanisme proposé
§ Depuis, plusieurs études ont ______ à reproduire cette
association

A

Calcium et santé cardiovasculaire

À la fin des années 2000, une étude a rapporté une _____ risque
d’infarctus du myocarde chez ceux qui prenaient des
suppléments de calcium
§ L’augmentation de la ______ vasculaire était le
mécanisme proposé
§ Depuis, plusieurs études ont ______ à reproduire cette
association

434
Q

Calcium et santé cardiovasculaire
Une _______ existe entre les symptômes gastro-intestinaux et
cardiaques chez les gens qui prennent des suppléments de calcium

Cette confusion a contribué à ________ les cas de symptômes
cardiaques dans les études où les participants auto-rapportaient
leurs symptômes

Lorsque les symptômes étaient évalués objectivement par l’équipe
médicale:
– Les symptômes gastro-intestinaux étaient plus ________
chez ceux qui prenaient des suppléments de calcium
– Les symptômes cardiaques n’étaient pas plus _______
chez ceux qui prenaient des suppléments de calcium

A

Une confusion existe entre les symptômes gastro-intestinaux et
cardiaques chez les gens qui prennent des suppléments de calcium

Cette confusion a contribué à surestimer les cas de symptômes
cardiaques dans les études où les participants auto-rapportaient
leurs symptômes

Lorsque les symptômes étaient évalués objectivement par l’équipe
médicale:
– Les symptômes gastro-intestinaux étaient plus fréquents
chez ceux qui prenaient des suppléments de calcium
– Les symptômes cardiaques n’étaient pas plus fréquents
chez ceux qui prenaient des suppléments de calcium

435
Q

Calcium et santé cardiovasculaire

Selon la position adoptée par The National Osteoporosis
Foundation and The American Society for Preventive
Cardiology (2016) aux Etats-Unis:
• « Des apports quotidiens en calcium (aliments et/ou
suppléments) ___ AMT sont considérés sécuritaires pour la
santé cardiovasculaire »

A

Selon la position adoptée par The National Osteoporosis
Foundation and The American Society for Preventive
Cardiology (2016) aux Etats-Unis:
• « Des apports quotidiens en calcium (aliments et/ou
suppléments) inférieur a l’AMT sont considérés sécuritaires pour la
santé cardiovasculaire »

436
Q

Suppléments de calcium (types)
Nommez les 3 types

Donnez des exemples pour chaque

A
Complexes
de calcium 
Carbonate (Tums)
Citrate
Lactate
Gluconate
« Calcium from bone meal»
Extraits
d’hydroxyapatite
et de collagène
d’os bovin

« Coral Calcium »
Calcium issu
des coraux

437
Q

Comment qualifie on le calcium from bone meal et le coral calcium et quel est le risuqe avec ces sources?

A

peut contenir contaminants plomb

438
Q

Pour les complexes de calcium, lequel a une meilleure biodisponibilité a jeun? pourquoi?

A
Pris à jeûn:
Citrate>carbonate
Citrate n’a pas besoin
de l’acidité gastrique
pour être absorbé
** Bon supplément
pour les achlorhydriques
439
Q

Pour les supplements de calcium, a quoi la quantité fait elle reference???

A

§ La quantité de calcium recommandée fait toujours référence
au calcium élémentaire
§ La quantité normalement indiquée sur les suppléments fait
référence au calcium élémentaire

440
Q

Supplements calcium

Vrai ou faux, Quantité indiquée n’est pas toujours la quantité de calcium contenue dans le comprimé

A

vrai

441
Q

Calcium: Effets secondaires possibles

quels sont les symptomes les plus rapportés?

A quoi sont ils liés?? peuvent ils arriver a une dose sous l’AMT?

A

Symptômes gastro-intestinaux sont les plus couramment rapportés:
§ Surtout constipation et gaz
§ Autres symptômes rapportés: mal de ventre, crampes, gonflements

Ces symptômes sont liés à la prise du supplément et peuvent survenir
même avec des apports < AMT

442
Q

supplements de calcium
Facteurs en prendre en considération:
Interactions…médicamenteuses!

Quel est l’effet en general du calcium?

Comment previent on ces effets nefastes?

QUel medicament a un effet différent et quelle est la conséquence?

A

DIminue efficacité medicament ou diminue absopriton.

prendre les repas espacé du medicament,

Digoxin ↑ activité
Si hypercalcémie
concomitante: ↑ risque
arythmie fatale

443
Q

Quelles sont les 2 précurseurs de la vitamine D? Quelle est leur différence?
QUel est leur numéro respectif et leur nom respectif une fois irradié?

A

Ergorstérol (seulement vegetal) D2, ergocalciférol

vs

7-déhydrocholesterol (Animal et vegetal), D3, Cholecalciférol

444
Q

Décrivez la photobiogenese de la 7 déhydro cholestérol

A

Dans la zone superficielles de la peau (derme et épiderme) les UVB la convertisse en pre vitamine D3, qui suite a 4-8h a 37 C devient la Vitamine D3, qui circule liée a une protéine

445
Q

Comment le temps pour faire de la vitamine D par photobiogenese varie-t-il?

QUel est l’ordre de grandeur de temps pour 1000 IU?

A

Augmente plus la peau est foncée (2 fois plus long pour les personnes de races noirs vs peau tres blanche)

Augmente moins la surface de peau est grande

en termes de minutes jusqua dizaine de minutes

446
Q

Peut-on s’intoxiquer
à la vitamine D si on
prend trop de soleil?
Pourquoi?

A

Non, le corps a cette capacité de réguler la synthese de vit D, lorsque la peau le sent, elle detourne la voie metabolique, il y auara la synthese de produit inactif. le corps s,autoregule,

on peut quand meme s’intoxiquer par voie orale par contre.

447
Q

Nommez les 11 facteurs qui influencent la photobiogenes de la vit D

A
1. creme solaire
2Age
3vetements
4ombrage
5pollution
6keratinisation peau
7institutionalisation
8saison 
9latitude
10heure du jour
11 . couleur peau (mélanine)
448
Q

Décrivez le schéma d’absoprtion et métabolisme de la vit D a voix haute

A

Diete et supplements, sont mycélisés, puis diffuse dans l’intestin. La majorité est intégrée au chylomichron, lpetite partie diffuse dans le sang.

Une fois dans le sang, une petite portion va au muscle et on pense que les obeses en auraient dans le tissu adipeux. Cest ici que la vit D faite par photobiogenese entre aussi.

DU sang, elle va au foie, ou elle est metabolisée en calcidiol. Principalement ensuite, elle va au rien, mais une partie va aussi aux tissus extra reinaux quand on en a assez de calcidiol. sinon ça va tout aux reins.

DAns le rein, le calcidiol est encoyre hydroxyler pour devenir le calcitriol, qui retourne au sang pour faire ses actions typiques endocrines.

Dans la voie des tissus extrareinaux, elle peut aussi etre hydroxylée, mais les actions seraient auto ou paracrines. Cest documenté dans certaines cellules. macrophages, les osteoblastes, et aussi on pense que la prostate, l’intestin, les glandes mammaires etc. le capte. bref, beaucoup de cellules.

449
Q

Décrivez la voie extra reinale de la vitamine D en utilisant l’Exemple du macrophage.

A

la tuberculose est un pathogene reconnu par le macrophage. TLR1 et 2 sont des recepteurs qui réagissent a la presence de pathogene. ça va activer a l’interieur du macrophage la voie extra reinale de la vit D. (donc l’hydoxylase et le receptemur a vitamine D nucleire.

L’interaciton du pathogene avec son recepteur va favoriser la synthese de l’enzyme . le macrophage doit etre stimulé pour synthetiser la vit D.

fonctionne un peu comme l’acide retinoique, va moduler l’expression de gene dont l’autophagie, c’est le processus qui detruit le pathogene. l’autophagosome detruit le pathogene.

l’autophagosome secrete aussi la cathelicidin qui est un peptide antimicrobien.

pour que ça se passe bien dans le macrophage, on doit avoir bcp de vit D dans le sang. ca nempeche pas le macrophage de tuer le pathogene si on ne la pas, mais le macrophage va etre moins efficace. la vit D donne un petit boost au macrophage pour l’aider.

450
Q

Régulation du métabolisme de la vitamine D

Décrivez le schéma de regulation. (QU’est-ce qui contribue a augmenter ou diminuer le metabolisme de la vit D)

A

On ne sait pas trop mais de gros apports pourrait avoir un impact.

Aussi si la vit D finit dans le tissu adipeux ca pourrait avoir un imapct

ensuite il y a la biophotogenese et tous les facteurs qui l’influencent.

La parathormone, phosphoremie, calcemie, influence la production de vit D.

AU niveau du foie, il y a retroaction negative du calcidiol sur sa production. Aussi trees genentique

451
Q

Certaines etudes ont montré que les obeses retenainet moins de vitamine D dans le serum qun groupe controle, comment a ton interprete cette etude?

A

personnes obeses ont une reponse sous optimale, la vit D serait séquestrée dans l’exces de tissu adipeux, mais cest un facteur qui na jamais été prouvé.

sinon, les personnes obeses ont plus de volume sanguin, si on donne une dose fixe, il y a de la dilution. ce serait plus un effet de dilution que de sequestration.

doit on donner plus de vit D aux personnes obeses?? il n’y a pas de recommandation encore.

452
Q

Décrivez l’impact d’une diminution de la calcémie sur la vitamine D (Schéma de regulation de la calcemie)

A

La diminution est percu par la parathyroide, qui secrete la PTH. Celle ci va augmente l’enzyme qui hydroxyle le calcidiol en calcitriol. Le calctriol augemnete l’Absoprtion de calcium et phosphore, favorise la resoprtion osseuse. La vit D favorise la conservation du calcium et phosphore dans le rein.

453
Q

Pourquoi la PTH exerce-t-elle
des actions inverses sur le
métabolisme du calcium et du
phosphore?

A

les deux ensembles, si on les augmente trop, ca atteint un niveau de saturaiton, ils vont complexer, et passé ce point, ça précipite. ça peut former des pierres. donc les 2 concentrations doivent etre bien régulées. donc quand on monte un on essaie de baisser l’autre. dans le contexte du sang, mais dans le squellette on veut augmenter les 2 pour aller minéraliser. mais ce nest pas la PTH qui regule ça ,elle regule la resorption pas la mineralisation.

454
Q

Explique comment l’action nucléique de la vit D se fait

A

ça prend la vit D active et lacide retinoique pour faire ça.

la reponse est une sequence de base nucléique sur l’ADN,

, ce sont les recepteurs RXR VDR, la D3 lie le VDR, l’acide retinoique lie le RXR, l’element de reponse est une sequence de base nucleiuqe sur lADN.

ça influence la transcription genique et ultimement la synthese de proteines.

3 proteines sont vitamine D dependantes dans l’enterocyte.

455
Q

Décrivez comment la vit D favorise le remodelage osseux

A
  1. qu’est-ce que la vit D fait?
    controle la calcemie et Phosphoremie, que c’est optimal et que ça sert bien a l’os.
  2. la vit D regule la synthese du collagene par l’osteoblaste. la collagene de type 1 est un gene vitamine D dépendant et la vit D controle donc la formation de l’os.

3, la vit D controle aussi le remodelage, fait secreter des cytokine pro-osteoclastique.

456
Q

quelles sont les 2 fonctions non classiques de la vit D?
Nommez au niveau de quelles cellules les actions non classiques de la vitamine D se font. Classez les selons qu’elles previennent le cancer, le psoriasis ou les maladies autoimmunes

A

ANti proliferation,
Pro differenciation

Cancer
Entérocytes
Colonocytes
Cellules sein
Cellules prostate
Psoriasis
Kératinocytes
Maladies autoimmunes
Lymphocytes T et B
457
Q

C’est quoi le psoriasis?

A

psoriaisis, hyperproliferation des cellules de la peau, on prescrit des cremes a base de vit D. traitement du psoriasis.

458
Q

Marqueur du statut
nutritionnel en vitamine D
PAR COEUR IMPORTANT

quelles sont les zones frontieres et les status associés (en nmol/L) de 25(OH)D

A

Zone de toxicité
250 nmol/L
zone de suffisance physiologique
80 nmol/L
zone de suffisance squelletique, sous optimale correct pour le squellette, mais pour les fonctions extrareinales cest encore sous optimale. donc on devrait se tenir au dessus de 80.
50 nmol/L
zone d’insuffisance Associée à des maladies de longue
latence
25 nmol/L
zone de déficience Associée à des signes cliniques
de carence: rachitisme/ostéomalacie

459
Q

Comment varie les ANREF de la vit D selon l’age, le sexe, et le cycle de vie

A

Pareil pour homme et femmes

nourissons ont moins besoins, puis augmente mais meme chose a partir de 9 ans

Grossess et Allaitement cest pareil

460
Q

Expliquex les variations des recommandations en vit D selon les instituts.

A

GAC recommande peu, institut de medecine plus, societe canadienne du cancer recommande 1000 meme si peu de données, osteoporose canada recommande de 1000 a 2000 dependanmment de lage et du risque ou du diagnostic, la endocrine society recommande 2000

461
Q

Vitamine D2 ou D3:
Est-ce du pareil au même
pour le corps?

A

quelqun qui consomme seulement la D2, le corps va l’utiliser, mais entre la d3 et la d2, le corps a une petite preference. nos enzymes ont une meilleure affinité pour la d3. mais si on a que de la d2 le corps l’utilise quand meme.

462
Q

Vit D

Comment peut on atteindre les recommandations de l’institut de medecine avec avec des aliments? a quoi doit on faire attention?

A

huile de foie de morue ou foie de morue

faire attenetion a la vit A ,surtout pour femme enceinte (foie contient bcp de vit A)

Sinon saumon

ou sources fortifiées ça en prend beaucoup.

463
Q

Quelle est la difference entre insuffisance en vit D et déficience?

A

Associée à des maladies de longue
latence

Associée à des signes cliniques
de carence: rachitisme/ostéomalacie

464
Q

A l’aide du schema du metabolisme de la vit D, décrivez ce qui se passe en cas de carence en vit D

A

la diminution de calcitriol fait quo absorbe moins de calcium et quon augmente la PTH.

La PTH augmente la resoprtion, et augmente le phosphore urinaire. La diminution du Ca et P serique reaugmente la PTH, qui reaugmente le phosphore urinaire. cest un cercle.

Los se déminéralise.

Au final ca fait le rachitisme ou l’ostéomalacie

465
Q

vair ou faux

L’ostéomalacie est considéré comme le “rachitisme” adulte

A

Faux

peut coexister chez l’enfant. (l’osteomalacie).

la distinction entre les 2, cest le site ou il y a le défaut, le rachitisme touche la plaque de croissance de l’os, ce qui fait grandir les os en longueurs. ca touche donc la plaque de croissance, qui est une structure de cartilage dans nos os longs, ce qui fait pousser nos os en longueur.

osteomalacie ne touche pas la xone de croissance, touche la zone en dessous, le site anatomique n’Est pas le meme, cest pour ça que ça peut coexister. osteomlacie toche la metaphyse, la partie mediane de l’os long,

en bout de ligne ca donne un retard de croissance mais les 2 nont pas le meme site d’action. Che xladulte, pas de plaque de croissance, donc pas de rachitisme.

466
Q

Le rachitisme et l’ostéomalacie sont des formes d’ostéoporose

A

faux, 3 entites cliniques distinctes, osteoporose nest pas un defaut de mineralisation, cest un probleme au niveau du remodelage osseux, ca tend vers la resoprtion osseuse, qui est plus importante, que la formation. on resorbe plus que former. on a donc des travees osseuses pas assez formée, ou pas assez rapidement, ce qui fait que l’os est fragile. defaut plus au niveau du remodelage osseux. oui la carence en vit D peut accentuer l’osteoporose, mais ca ne veut pas dire que cest la cause. osteoporose peut etre causé par plein d’autres choses.
Osteoporose est elle necessairement associé a lage? non peut etre aussi lié a l’anoerexie, maladie de crohn, (osteoporose secondaire), pas juste lié au vieillissement ou a la menopause.

467
Q

Vit D

Populations à risque

1) Enfants exclusivement allaités

Pourquoi?

A

carence d’apports. (pas assez dans le lait maternel). (25 UI par L)

468
Q

Vit D

Populations à risque

2) Enfants qui ne sortent pas ou de régions
peu ensoleillées

Pourquoi?

A

photobiogenese est compromise. (2 et 3 donc elimination dune sourc eimportante de vit D)

469
Q

Vit D

Populations à risque

3) Enfants qui sont couverts pour des
raisons culturelles/climatiques

Pourquoi?

A

photobiogenese est compromise. (2 et 3 donc elimination dune sourc eimportante de vit D)

470
Q

Vit D

Populations à risque

4) Individus ayant des pathologies
hépatiques

Pourquoi?

A

le foie produit le calcidiol (25 hydroxy)vitD, si il ne produit pas ac peut etre un probleme

dans une moindre mesure, la prot de transport de la vit D.

ou probleme et production de bile (ou ecoulement de bile) on en fait pas les micelles, ca va impacter l’abosrption.

471
Q

Vit D

Populations à risque

5) Individus ayant des pathologies
intestinales

Pourquoi?

A

nuit a l,absoprtion

472
Q

Vit D

Populations à risque

6) Individus ayant des pathologies rénales

Pourquoi?

A

le rein fait la 1,25, si cest impacté, on ne la fait pas. ces gens la on leur donne carréement la 1,25.

473
Q

Vit D

Populations à risque

7) Individus ayant des atteintes cutanées
sévères

Pourquoi?

A

ne font pas la photobiogenese.

474
Q

Vit D

Chez l’enfant, quels sont les signes physiques de carence en vit D?

A

bosses frontales, defaut de mineralisation de l’os du crane.

bourlet hepiphysaire au niveau des poignets.

chapelets costaux, boule sur les cotes, signes de défauts de mineralisation.

475
Q

Quel est le signe histologique du rachitisme?

A

Élargissement
Plaque de croissance
épiphysaire indique
un défaut de minéralisation

476
Q

vrai ou faux

L’ostéomalacie est difficile à diagnostiquer cliniquement
en raison de l’absence de signes physiques

A

vrai

477
Q

Comment peut on diagnostiquer l’osteomlacie a l’aide d’une biopsie?

A

ici biopsie de l’os (quand on soupconne) on doit faire une anasthesie locale . elle est colorée (travée ossesuse)

rouge est collagene, bleu vert est minéralisé. Plus cest rouge, moins cest minéralisé (collagene pas minéralisé)”

Donc on a une matrice mais il ny a pas de minéraux dedans.

on quantifie les rapports des surfaces et on determine quil y a osteomalacie ou pas.

478
Q

Quelle conséquence la carence en vit D peut avoir sur les:

jambes?

Colonne vertebrale?

Reproduction des femmes?

A

Arquée

Scoliose

Consequence sur l’os pelvien ,empechera de procréer

479
Q

Vrai ou faux ?
La déficience en vitamine D est la seule carence
nutritionnelle qui mène au rachitisme

A

faux, carence en calcium ou phosphore peut aussi donner un rachitisme.

480
Q

Les rachitismes « non-nutritionnels »

Donnez un exemple de rachitisme non nutritionnel et le symptome

A

maladie genetique, ici, personnes ont une mutation dans le VDR (recepteur vit D), il est non fonctionnel, ne repond pas a la 1,25. rachitisme osseux, et les poils ne poussent pas, donc la vit D aurait une action a ce niveau la. cest un rachitisme genetique.

aussi, certains nont pas l’enzyme reinal (1,25 hydroxylase). on peut l’administrer, dans ce cas la.

Mais si on a pas de VDR, on peut donner du calcium et phosphore pour s’assurer que la mineralisation osseuse se fasse. mais la vit D aide a l’abosrption. on doit le donner intraveineux car oralement l’intestin nabsorberait pas meme a tres grande dose. souvent sera des infusions de calcium, le phosphore est supplémenté dune autre façon.

481
Q

Quel est le critere pour l’AMT de la vit D?

A

hyperabsoprtion de calcium, hypercalcemie, hypercalciurie, risque de calcification des tissus mous.

482
Q

Comment l’AMT de la vit D varie pour le sexe?

Pour l’age?

Pour le cycle de vie?

A

Varie pas

Augmente progressivement, est maximal a partir de 9 ans et pour la grossesse et l’allaitement

483
Q

D’ou vient le risque de trop consommer de la vit D? (se referer au metabolisme de la vit D)

A

Viendrait du calcidiol, lorsque en grand exces, favorise aussi la resoprtion de l’os, l’hypercalcémie qui peut créer de la calcification vasculaire ou des tissus mou ou eventuellement des nephrolithiases